Problem with AnyConnect VPN on ASA 5505

Hello everyone,
We're troubleshooting an issue where a client cannot pass any traffic across an AnyConnect VPN with an ASA5505 as the endpoint. The client receives and IP address in the 172.16.0.1/24 range and the ASA creates a static route to the 10.0.0.0/24 internal network but we cannot ping or connect to any internal IP address. The connection appears to fully build and pass traffic (based on the byte counts which increase) but we can't talk to the main network.
Does anyone have any ideas as to what I can check?
Thanks!
Ryan

AnyConnect client should not be in the same subnet as the internal hosts. It needs to be unique subnet within your environment, so you were on the right path initially.
If you can share your config, that would be easier for us to check.
In the meantime, a few things to check:
1) Have you configured split tunnel policy?
2) Do you have NAT exemption configured?
3) Any VPN filter configured that might be blocking the traffic?
4) Does the internal network know how to route back to the VPN Pool subnet (ie: via the ASA)
5) Lastly, do you have "inspect icmp" configured?

Similar Messages

  • Problem with saving config on asa 5505

    I have asa 5505 with 512mb , i am trying to save ios on it but i failed many times , I have sandisk 2GB and i formatted with windows on fat but every time i see disk0 failed , any ideas what is the problem ?
    Thanks                  

    I have asa 5505 with 512mb , i am trying to save ios on it but i failed many times , I have sandisk 2GB and i formatted with windows on fat but every time i see disk0 failed , any ideas what is the problem ?
    Thanks                  

  • Site to Site VPN Problems With 2801 Router and ASA 5505

    Hello,
    I am having some issue setting up a site to site ipsec VPN between a Cisco 2801 router and a Cisco ASA 5505. I was told there was a vpn previously setup with an old hosting provider, but those connections have been servered. Right now I am trying to get the sites to talk to the 2801. Here ere are my current configs, please let me know if you need anything else. Im stumped on this one. Thanks.
    IP scheme at SIte A:
    IP    172.19.3.x
    sub 255.255.255.128
    GW 172.19.3.129
    Site A Ciscso 2801 Router
    Current configuration : 11858 bytes
    version 12.4
    service timestamps debug datetime localtime
    service timestamps log datetime localtime show-timezone
    service password-encryption
    hostname router-2801
    boot-start-marker
    boot-end-marker
    logging message-counter syslog
    logging buffered 4096
    aaa new-model
    aaa authentication login userauthen group radius local
    aaa authorization network groupauthor local
    aaa session-id common
    clock timezone est -5
    clock summer-time zone recurring last Sun Mar 2:00 1 Sun Nov 2:00
    dot11 syslog
    ip source-route
    ip dhcp excluded-address 172.19.3.129 172.19.3.149
    ip dhcp excluded-address 172.19.10.1 172.19.10.253
    ip dhcp excluded-address 172.19.3.140
    ip dhcp ping timeout 900
    ip dhcp pool DHCP
       network 172.19.3.128 255.255.255.128
       default-router 172.19.3.129
       domain-name domain.local
       netbios-name-server 172.19.3.7
       option 66 ascii 172.19.3.225
       dns-server 172.19.3.140 208.67.220.220 208.67.222.222
    ip dhcp pool VoiceDHCP
       network 172.19.10.0 255.255.255.0
       default-router 172.19.10.1
       dns-server 208.67.220.220 8.8.8.8
       option 66 ascii 172.19.10.2
       lease 2
    ip cef
    ip inspect name SDM_LOW cuseeme
    ip inspect name SDM_LOW dns
    ip inspect name SDM_LOW ftp
    ip inspect name SDM_LOW h323
    ip inspect name SDM_LOW https
    ip inspect name SDM_LOW icmp
    ip inspect name SDM_LOW imap
    ip inspect name SDM_LOW pop3
    ip inspect name SDM_LOW netshow
    ip inspect name SDM_LOW rcmd
    ip inspect name SDM_LOW realaudio
    ip inspect name SDM_LOW rtsp
    ip inspect name SDM_LOW esmtp
    ip inspect name SDM_LOW sqlnet
    ip inspect name SDM_LOW streamworks
    ip inspect name SDM_LOW tftp
    ip inspect name SDM_LOW tcp
    ip inspect name SDM_LOW udp
    ip inspect name SDM_LOW vdolive
    no ip domain lookup
    ip domain name domain.local
    multilink bundle-name authenticated
    key chain key1
    key 1
       key-string 7 06040033484B1B484557
    crypto pki trustpoint TP-self-signed-3448656681
    enrollment selfsigned
    subject-name cn=IOS-Self-Signed-Certificate-3448bb6681
    revocation-check none
    rsakeypair TP-self-signed-344bbb56681
    crypto pki certificate chain TP-self-signed-3448656681
    certificate self-signed 01
      3082024F
                quit
    username admin privilege 15 password 7 F55
    archive
    log config
      hidekeys
    crypto isakmp policy 10
    encr 3des
    hash md5
    authentication pre-share
    group 2
    crypto isakmp key XXXXX address 209.118.0.1
    crypto isakmp key xxxxx address SITE B Public IP
    crypto isakmp keepalive 40 5
    crypto isakmp nat keepalive 20
    crypto isakmp client configuration group IISVPN
    key 1nsur3m3
    dns 172.19.3.140
    wins 172.19.3.140
    domain domain.local
    pool VPN_Pool
    acl 198
    crypto isakmp profile IISVPNClient
       description VPN clients profile
       match identity group IISVPN
       client authentication list userauthen
       isakmp authorization list groupauthor
       client configuration address respond
    crypto ipsec transform-set myset esp-3des esp-md5-hmac
    crypto dynamic-map Dynamic 5
    set transform-set myset
    set isakmp-profile IISVPNClient
    qos pre-classify
    crypto map VPN 10 ipsec-isakmp
    set peer 209.118.0.1
    set peer SITE B Public IP
    set transform-set myset
    match address 101
    qos pre-classify
    crypto map VPN 65535 ipsec-isakmp dynamic Dynamic
    track 123 ip sla 1 reachability
    delay down 15 up 10
    class-map match-any VoiceTraffic
    match protocol rtp audio
    match protocol h323
    match protocol rtcp
    match access-group name VOIP
    match protocol sip
    class-map match-any RDP
    match access-group 199
    policy-map QOS
    class VoiceTraffic
        bandwidth 512
    class RDP
        bandwidth 768
    policy-map MainQOS
    class class-default
        shape average 1500000
      service-policy QOS
    interface FastEthernet0/0
    description $ETH-LAN$$ETH-SW-LAUNCH$$INTF-INFO-FE 0$$FW_INSIDE$
    ip address 172.19.3.129 255.255.255.128
    ip access-group 100 in
    ip inspect SDM_LOW in
    ip nat inside
    ip virtual-reassembly
    duplex auto
    speed auto
    interface FastEthernet0/0.10
    description $ETH-VoiceVLAN$$
    encapsulation dot1Q 10
    ip address 172.19.10.1 255.255.255.0
    ip inspect SDM_LOW in
    ip nat inside
    ip virtual-reassembly
    interface FastEthernet0/1
    description "Comcast"
    ip address PUB IP 255.255.255.248
    ip access-group 102 in
    ip inspect SDM_LOW out
    ip nat outside
    ip virtual-reassembly
    duplex auto
    speed auto
    crypto map VPN
    interface Serial0/1/0
    description "Verizon LEC Circuit ID: w0w13908 Site ID: U276420-1"
    bandwidth 1536
    no ip address
    encapsulation frame-relay IETF
    frame-relay lmi-type ansi
    interface Serial0/1/0.1 point-to-point
    bandwidth 1536
    ip address 152.000.000.18 255.255.255.252
    ip access-group 102 in
    ip verify unicast reverse-path
    ip inspect SDM_LOW out
    ip nat outside
    ip virtual-reassembly
    frame-relay interface-dlci 500 IETF 
    crypto map VPN
    service-policy output MainQOS
    interface Serial0/2/0
    description "PAETEC 46.HCGS.788446.CV (Verizon ID) / 46.HCGS.3 (PAETEC ID)"
    ip address 123.252.123.102 255.255.255.252
    ip access-group 102 in
    ip inspect SDM_LOW out
    ip nat outside
    ip virtual-reassembly
    encapsulation ppp
    crypto map VPN
    service-policy output MainQOS
    ip local pool VPN_Pool 172.20.3.130 172.20.3.254
    ip forward-protocol nd
    ip route 0.0.0.0 0.0.0.0 50.00.000.110 track 123
    ip route 0.0.0.0 0.0.0.0 111.252.237.000 254
    ip route 122.112.197.20 255.255.255.255 209.252.237.101
    ip route 208.67.220.220 255.255.255.255 50.78.233.110
    no ip http server
    no ip http secure-server
    ip http timeout-policy idle 60 life 86400 requests 10000
    ip flow-top-talkers
    top 20
    sort-by bytes
    ip nat inside source route-map COMCAST interface FastEthernet0/1 overload
    ip nat inside source route-map PAETEC interface Serial0/2/0 overload
    ip nat inside source route-map VERIZON interface Serial0/1/0.1 overload
    ip nat inside source static tcp 172.19.3.140 21 PUB IP 21 extendable
    ip access-list extended VOIP
    permit ip 172.20.3.0 0.0.0.127 host 172.19.3.190
    permit ip host 172.19.3.190 172.20.3.0 0.0.0.127
    ip radius source-interface FastEthernet0/0
    ip sla 1
    icmp-echo 000.67.220.220 source-interface FastEthernet0/1
    timeout 10000
    frequency 15
    ip sla schedule 1 life forever start-time now
    access-list 23 permit 172.19.3.0 0.0.0.127
    access-list 23 permit 172.19.3.128 0.0.0.127
    access-list 23 permit 173.189.251.192 0.0.0.63
    access-list 23 permit 107.0.197.0 0.0.0.63
    access-list 23 permit 173.163.157.32 0.0.0.15
    access-list 23 permit 72.55.33.0 0.0.0.255
    access-list 23 permit 172.19.5.0 0.0.0.63
    access-list 100 remark "Outgoing Traffic"
    access-list 100 deny   ip 67.128.87.156 0.0.0.3 any
    access-list 100 deny   ip host 255.255.255.255 any
    access-list 100 deny   ip 127.0.0.0 0.255.255.255 any
    access-list 100 permit tcp host 172.19.3.190 any eq smtp
    access-list 100 permit tcp host 172.19.3.137 any eq smtp
    access-list 100 permit tcp any host 66.251.35.131 eq smtp
    access-list 100 permit tcp any host 173.201.193.101 eq smtp
    access-list 100 permit ip any any
    access-list 100 permit tcp any any eq ftp
    access-list 101 remark "Interesting VPN Traffic"
    access-list 101 permit ip 172.19.3.128 0.0.0.127 172.19.3.0 0.0.0.127
    access-list 101 permit ip 172.20.3.128 0.0.0.127 172.19.3.0 0.0.0.127
    access-list 101 permit ip 172.19.3.128 0.0.0.127 host 172.19.250.10
    access-list 101 permit ip 172.19.3.128 0.0.0.127 host 172.19.250.11
    access-list 101 permit tcp any any eq ftp
    access-list 101 permit tcp any any eq ftp-data
    access-list 102 remark "Inbound Access"
    access-list 102 permit udp any host 152.179.53.18 eq non500-isakmp
    access-list 102 permit udp any host 152.179.53.18 eq isakmp
    access-list 102 permit esp any host 152.179.53.18
    access-list 102 permit ahp any host 152.179.53.18
    access-list 102 permit udp any host 209.000.000.102 eq non500-isakmp
    access-list 102 permit udp any host 209.000.000.102 eq isakmp
    access-list 102 permit esp any host 209.000.000.102
    access-list 102 permit ahp any host 209.000.000.102
    access-list 102 permit udp any host PUB IP eq non500-isakmp
    access-list 102 permit udp any host PUB IP eq isakmp
    access-list 102 permit esp any host PUB IP
    access-list 102 permit ahp any host PUB IP
    access-list 102 permit ip 72.55.33.0 0.0.0.255 any
    access-list 102 permit ip 107.0.197.0 0.0.0.63 any
    access-list 102 deny   ip 172.19.3.128 0.0.0.127 any
    access-list 102 permit icmp any any echo-reply
    access-list 102 permit icmp any any time-exceeded
    access-list 102 permit icmp any any unreachable
    access-list 102 permit icmp any any
    access-list 102 deny   ip any any log
    access-list 102 permit tcp any host 172.19.3.140 eq ftp
    access-list 102 permit tcp any host 172.19.3.140 eq ftp-data established
    access-list 102 permit udp any host SITE B Public IP  eq non500-isakmp
    access-list 102 permit udp any host SITE B Public IP  eq isakmp
    access-list 102 permit esp any host SITE B Public IP
    access-list 102 permit ahp any host SITE B Public IP
    access-list 110 remark "Outbound NAT Rule"
    access-list 110 remark "Deny VPN Traffic NAT"
    access-list 110 deny   ip 172.19.3.128 0.0.0.127 172.19.3.0 0.0.0.127
    access-list 110 deny   ip 172.19.3.128 0.0.0.127 172.19.10.0 0.0.0.255
    access-list 110 deny   ip 172.19.10.0 0.0.0.255 172.19.3.128 0.0.0.127
    access-list 110 deny   ip 172.20.3.128 0.0.0.127 172.19.3.0 0.0.0.127
    access-list 110 deny   ip 172.19.3.128 0.0.0.127 172.20.3.128 0.0.0.127
    access-list 110 deny   ip 172.19.3.128 0.0.0.127 host 172.19.250.11
    access-list 110 deny   ip 172.19.3.128 0.0.0.127 host 172.19.250.10
    access-list 110 permit ip 172.19.3.128 0.0.0.127 any
    access-list 110 permit ip 172.19.10.0 0.0.0.255 any
    access-list 198 remark "Networks for IISVPN Client"
    access-list 198 permit ip 172.19.3.0 0.0.0.127 172.20.3.128 0.0.0.127
    access-list 198 permit ip 172.19.3.128 0.0.0.127 172.20.3.128 0.0.0.127
    access-list 199 permit tcp any any eq 3389
    route-map PAETEC permit 10
    match ip address 110
    match interface Serial0/2/0
    route-map COMCAST permit 10
    match ip address 110
    match interface FastEthernet0/1
    route-map VERIZON permit 10
    match ip address 110
    match interface Serial0/1/0.1
    snmp-server community 123 RO
    radius-server host 172.19.3.7 auth-port 1645 acct-port 1646 key 7 000000000000000
    control-plane
    line con 0
    line aux 0
    line vty 0 4
    access-class 23 in
    privilege level 15
    transport input telnet ssh
    line vty 5 15
    access-class 23 in
    privilege level 15
    transport input telnet ssh
    scheduler allocate 20000 1000
    ntp server 128.118.25.3
    ntp server 217.150.242.8
    end
    IP scheme at site B:
    ip     172.19.5.x
    sub  255.255.255.292
    gw   172.19.5.65
    Cisco ASA 5505 at Site B
    ASA Version 8.2(5)
    hostname ASA5505
    domain-name domain.com
    enable password b04DSH2HQqXwS8wi encrypted
    passwd b04DSH2HQqXwS8wi encrypted
    names
    interface Ethernet0/0
    switchport access vlan 2
    interface Ethernet0/1
    interface Ethernet0/2
    interface Ethernet0/3
    interface Ethernet0/4
    interface Ethernet0/5
    interface Ethernet0/6
    interface Ethernet0/7
    interface Vlan1
    nameif inside
    security-level 100
    ip address 172.19.5.65 255.255.255.192
    interface Vlan2
    nameif outside
    security-level 0
    ip address SITE B public IP 255.255.255.224
    boot system disk0:/asa825-k8.bin
    ftp mode passive
    clock timezone est -5
    clock summer-time zone recurring last Sun Mar 2:00 last Sun Oct 2:00
    dns server-group DefaultDNS
    domain-name iis-usa.com
    same-security-traffic permit intra-interface
    object-group network old hosting provider
    network-object 72.55.34.64 255.255.255.192
    network-object 72.55.33.0 255.255.255.0
    network-object 173.189.251.192 255.255.255.192
    network-object 173.163.157.32 255.255.255.240
    network-object 66.11.1.64 255.255.255.192
    network-object 107.0.197.0 255.255.255.192
    object-group network old hosting provider
    network-object host 172.19.250.10
    network-object host 172.19.250.11
    access-list 100 extended permit ip 172.19.5.64 255.255.255.192 object-group old hosting provider
    access-list 100 extended permit ip 172.19.5.64 255.255.255.192 172.19.3.128 255.255.255.128
    access-list 10 extended deny ip 0.0.0.0 255.0.0.0 any
    access-list 10 extended deny ip 127.0.0.0 255.0.0.0 any
    access-list 10 extended deny ip 169.254.0.0 255.255.0.0 any
    access-list 10 extended deny ip 172.16.0.0 255.255.0.0 any
    access-list 10 extended deny ip 224.0.0.0 224.0.0.0 any
    access-list 10 extended permit icmp any any echo-reply
    access-list 10 extended permit icmp any any time-exceeded
    access-list 10 extended permit icmp any any unreachable
    access-list 10 extended permit icmp any any traceroute
    access-list 10 extended permit icmp any any source-quench
    access-list 10 extended permit icmp any any
    access-list 10 extended permit tcp object-group old hosting provider any eq 3389
    access-list 10 extended permit tcp any any eq https
    access-list 10 extended permit tcp any any eq www
    access-list 110 extended permit ip 172.19.5.64 255.255.255.192 172.19.3.0 255.255.255.128
    access-list 110 extended permit ip 172.19.5.64 255.255.255.192 object-group old hosting provider
    pager lines 24
    logging enable
    logging timestamp
    logging console emergencies
    logging monitor emergencies
    logging buffered warnings
    logging trap debugging
    logging history debugging
    logging asdm informational
    mtu inside 1500
    mtu outside 1500
    ip verify reverse-path interface inside
    ip verify reverse-path interface outside
    ip audit name jab attack action alarm drop reset
    ip audit name probe info action alarm drop reset
    ip audit interface outside probe
    ip audit interface outside jab
    ip audit info action alarm drop reset
    ip audit attack action alarm drop reset
    ip audit signature 2000 disable
    ip audit signature 2001 disable
    ip audit signature 2004 disable
    ip audit signature 2005 disable
    icmp unreachable rate-limit 1 burst-size 1
    icmp permit 75.150.169.48 255.255.255.240 outside
    icmp permit 72.44.134.16 255.255.255.240 outside
    icmp permit 72.55.33.0 255.255.255.0 outside
    icmp permit any outside
    icmp permit 173.163.157.32 255.255.255.240 outside
    icmp permit 107.0.197.0 255.255.255.192 outside
    icmp permit 66.11.1.64 255.255.255.192 outside
    icmp deny any outside
    asdm image disk0:/asdm-645.bin
    no asdm history enable
    arp timeout 14400
    global (outside) 1 interface
    nat (inside) 0 access-list 100
    nat (inside) 1 0.0.0.0 0.0.0.0
    access-group 10 in interface outside
    route outside 0.0.0.0 0.0.0.0 174.78.151.225 1
    timeout xlate 3:00:00
    timeout conn 24:00:00 half-closed 0:10:00 udp 0:10:00 icmp 0:00:02
    timeout sunrpc 0:10:00 h323 0:05:00 h225 1:00:00 mgcp 0:05:00 mgcp-pat 0:05:00
    timeout sip 24:00:00 sip_media 0:02:00 sip-invite 0:03:00 sip-disconnect 0:02:00
    timeout sip-provisional-media 0:02:00 uauth 0:05:00 absolute
    timeout tcp-proxy-reassembly 0:01:00
    timeout floating-conn 0:00:00
    dynamic-access-policy-record DfltAccessPolicy
    http 107.0.197.0 255.255.255.192 outside
    http 66.11.1.64 255.255.255.192 outside
    snmp-server host outside 107.0.197.29 community *****
    snmp-server host outside 107.0.197.30 community *****
    snmp-server host inside 172.19.250.10 community *****
    snmp-server host outside 172.19.250.10 community *****
    snmp-server host inside 172.19.250.11 community *****
    snmp-server host outside 172.19.250.11 community *****
    snmp-server host outside 68.82.122.239 community *****
    snmp-server host outside 72.55.33.37 community *****
    snmp-server host outside 72.55.33.38 community *****
    snmp-server host outside 75.150.169.50 community *****
    snmp-server host outside 75.150.169.51 community *****
    no snmp-server location
    no snmp-server contact
    snmp-server community *****
    snmp-server enable traps snmp authentication linkup linkdown coldstart
    crypto ipsec transform-set ESP-3DES-MD5 esp-3des esp-md5-hmac
    crypto ipsec security-association lifetime seconds 28800
    crypto ipsec security-association lifetime kilobytes 4608000
    crypto map VPNMAP 10 match address 110
    crypto map VPNMAP 10 set peer 72.00.00.7 old vpn public ip Site B Public IP
    crypto map VPNMAP 10 set transform-set ESP-3DES-MD5
    crypto map VPNMAP 10 set security-association lifetime seconds 86400
    crypto map VPNMAP 10 set security-association lifetime kilobytes 4608000
    crypto map VPNMAP interface outside
    crypto isakmp identity address
    crypto isakmp enable outside
    crypto isakmp policy 20
    authentication pre-share
    encryption 3des
    hash md5
    group 2
    lifetime 86400
    telnet 172.19.5.64 255.255.255.192 inside
    telnet 172.19.3.0 255.255.255.128 outside
    telnet timeout 60
    ssh 0.0.0.0 0.0.0.0 inside
    ssh 0.0.0.0 0.0.0.0 outside
    ssh timeout 60
    console timeout 0
    management-access inside
    dhcpd dns 172.19.3.140
    dhcpd wins 172.19.3.140
    dhcpd ping_timeout 750
    dhcpd domain iis-usa.com
    dhcpd address 172.19.5.80-172.19.5.111 inside
    dhcpd enable inside
    threat-detection basic-threat
    threat-detection scanning-threat shun except object-group old hosting provider
    threat-detection statistics
    threat-detection statistics tcp-intercept rate-interval 30 burst-rate 400 average-rate 200
    ntp server 128.118.25.3 source outside
    ntp server 217.150.242.8 source outside
    tunnel-group 72.00.00.7 type ipsec-l2l
    tunnel-group 72.00.00.7 ipsec-attributes
    pre-shared-key *****
    tunnel-group old vpn public ip type ipsec-l2l
    tunnel-group old vpn public ip ipsec-attributes
    pre-shared-key *****
    tunnel-group SITE A Public IP  type ipsec-l2l
    tunnel-group SITE A Public IP  ipsec-attributes
    pre-shared-key *****
    class-map inspection_default
    match default-inspection-traffic
    policy-map type inspect dns preset_dns_map
    parameters
      message-length maximum 512
    policy-map global_policy
    class inspection_default
      inspect dns preset_dns_map
      inspect ftp
      inspect h323 h225
      inspect h323 ras
      inspect rsh
      inspect rtsp
      inspect esmtp
      inspect sqlnet
      inspect skinny 
      inspect sunrpc
      inspect xdmcp
      inspect netbios
      inspect tftp
      inspect pptp
      inspect sip 
    service-policy global_policy global
    prompt hostname context
    no call-home reporting anonymous
    call-home
    profile CiscoTAC-1
      no active
      destination address http https://tools.cisco.com/its/service/oddce/services/DDCEService
      destination address email [email protected]
      destination transport-method http
      subscribe-to-alert-group diagnostic
      subscribe-to-alert-group environment
      subscribe-to-alert-group inventory periodic monthly
      subscribe-to-alert-group configuration periodic monthly
      subscribe-to-alert-group telemetry periodic daily
    Cryptochecksum:
    : end

    I have removed the old "set peer" and have added:
    IOS router:
    access-list 101 permit ip 172.19.3.128 0.0.0.127 172.19.5.64 0.0.0.65
    ASA fw:
    access-list 110 extended permit ip 172.19.5.64 255.255.255.192 172.19.3.128 255.255.255.128
    on the router I have also added;
    access-list 110 deny  ip 172.19.3.128 0.0.0.127 172.19.5.64 0.0.0.63
    Here is my acl :
    access-list 110 remark "Outbound NAT Rule"
    access-list 110 remark "Deny VPN Traffic NAT"
    access-list 110 deny   ip 172.19.3.128 0.0.0.127 172.19.3.0 0.0.0.127
    access-list 110 deny   ip 172.19.3.128 0.0.0.127 172.19.10.0 0.0.0.255
    access-list 110 deny   ip 172.19.10.0 0.0.0.255 172.19.3.128 0.0.0.127
    access-list 110 deny   ip 172.20.3.128 0.0.0.127 172.19.3.0 0.0.0.127
    access-list 110 deny   ip 172.19.3.128 0.0.0.127 172.20.3.128 0.0.0.127
    access-list 110 deny   ip 172.19.3.128 0.0.0.127 host 172.19.250.11
    access-list 110 deny   ip 172.19.3.128 0.0.0.127 host 172.19.250.10
    access-list 110 permit ip 172.19.3.128 0.0.0.127 any
    access-list 110 permit ip 172.19.10.0 0.0.0.255 any
    access-list 110 deny   ip 172.19.3.128 0.0.0.127 172.19.5.64 0.0.0.63
    access-list 198 remark "Networks for IISVPN Client"
    access-list 198 permit ip 172.19.3.0 0.0.0.127 172.20.3.128 0.0.0.127
    access-list 198 permit ip 172.19.3.128 0.0.0.127 172.20.3.128 0.0.0.127
    Still no ping tothe other site.

  • Problem with Configuring ACL on ASA 5505

    Dear All,
    i am trying to configure access list on asa 5505
    i have three interfaces
    guest with dhcp server
    inside static ip range
    outside internet
    i am trying to close the http protocol from some users in ( inside ) int by writing those command
    access-list OUT extended deny tcp host 172.16.100.197 any eq http
    access -group OUT out interface outside
    it's working but on all the inside network 172.16.100.0/22
    could you please help me to apply that on only the specific host or to create a group of hosts and assign this acl on it ?
    thanx all
    [BEGIN] 7/17/2012 5:31:15 PM
    sho run
    : Saved
    ASA Version 8.2(5)
    hostname ConcordeASA
    enable password 8Ry2YjIyt7RRXU24 encrypted
    passwd 2KFQnbNIdI.2KYOU encrypted
    names
    interface Ethernet0/0
    switchport access vlan 2
    interface Ethernet0/1
    interface Ethernet0/2
    interface Ethernet0/3
    interface Ethernet0/4
    switchport access vlan 12
    interface Ethernet0/5
    switchport access vlan 12
    interface Ethernet0/6
    <--- More --->
    interface Ethernet0/7
    interface Vlan1
    nameif inside
    security-level 100
    ip address 172.16.100.1 255.255.252.0
    interface Vlan2
    nameif outside
    security-level 0
    ip address 1.1.1.1 255.255.255.248
    interface Vlan12
    no forward interface Vlan1
    nameif Guest
    security-level 50
    ip address 192.168.1.1 255.255.252.0
    ftp mode passive
    dns domain-lookup inside
    dns domain-lookup outside
    dns domain-lookup Guest
    dns server-group DefaultDNS
    <--- More --->
    name-server 212.77.192.59
    name-server 212.77.192.60
    same-security-traffic permit intra-interface
    object-group protocol TCPUDP
    protocol-object udp
    protocol-object tcp
    access-list inside_access_in extended permit ip any any
    access-list Guest_access_in extended permit ip any any
    pager lines 24
    logging asdm informational
    mtu inside 1500
    mtu outside 1500
    mtu Guest 1500
    icmp unreachable rate-limit 1 burst-size 1
    no asdm history enable
    arp timeout 14400
    global (outside) 1 interface
    nat (inside) 1 0.0.0.0 0.0.0.0
    nat (Guest) 1 0.0.0.0 0.0.0.0
    access-group Guest_access_in in interface Guest
    route outside 0.0.0.0 0.0.0.0 78.100.85.250 1
    route inside 172.16.100.0 255.255.252.0 172.16.100.1 1
    timeout xlate 3:00:00
    timeout conn 1:00:00 half-closed 0:10:00 udp 0:02:00 icmp 0:00:02
    <--- More --->
    timeout sunrpc 0:10:00 h323 0:05:00 h225 1:00:00 mgcp 0:05:00 mgcp-pat 0:05:00
    timeout sip 0:30:00 sip_media 0:02:00 sip-invite 0:03:00 sip-disconnect 0:02:00
    timeout sip-provisional-media 0:02:00 uauth 0:05:00 absolute
    timeout tcp-proxy-reassembly 0:01:00
    timeout floating-conn 0:00:00
    dynamic-access-policy-record DfltAccessPolicy
    http server enable
    http 172.16.100.0 255.255.252.0 inside
    no snmp-server location
    no snmp-server contact
    snmp-server enable traps snmp authentication linkup linkdown coldstart
    crypto ipsec security-association lifetime seconds 28800
    crypto ipsec security-association lifetime kilobytes 4608000
    crypto ca trustpoint _SmartCallHome_ServerCA
    crl configure
    crypto ca certificate chain _SmartCallHome_ServerCA
    certificate ca 6ecc7aa5a7032009b8cebcf4e952d491
    308205ec 308204d4 a0030201 0202106e cc7aa5a7 032009b8 cebcf4e9 52d49130
    0d06092a 864886f7 0d010105 05003081 ca310b30 09060355 04061302 55533117
    30150603 55040a13 0e566572 69536967 6e2c2049 6e632e31 1f301d06 0355040b
    13165665 72695369 676e2054 72757374 204e6574 776f726b 313a3038 06035504
    0b133128 63292032 30303620 56657269 5369676e 2c20496e 632e202d 20466f72
    20617574 686f7269 7a656420 75736520 6f6e6c79 31453043 06035504 03133c56
    65726953 69676e20 436c6173 73203320 5075626c 69632050 72696d61 72792043
    <--- More --->
    65727469 66696361 74696f6e 20417574 686f7269 7479202d 20473530 1e170d31
    30303230 38303030 3030305a 170d3230 30323037 32333539 35395a30 81b5310b
    30090603 55040613 02555331 17301506 0355040a 130e5665 72695369 676e2c20
    496e632e 311f301d 06035504 0b131656 65726953 69676e20 54727573 74204e65
    74776f72 6b313b30 39060355 040b1332 5465726d 73206f66 20757365 20617420
    68747470 733a2f2f 7777772e 76657269 7369676e 2e636f6d 2f727061 20286329
    3130312f 302d0603 55040313 26566572 69536967 6e20436c 61737320 33205365
    63757265 20536572 76657220 4341202d 20473330 82012230 0d06092a 864886f7
    0d010101 05000382 010f0030 82010a02 82010100 b187841f c20c45f5 bcab2597
    a7ada23e 9cbaf6c1 39b88bca c2ac56c6 e5bb658e 444f4dce 6fed094a d4af4e10
    9c688b2e 957b899b 13cae234 34c1f35b f3497b62 83488174 d188786c 0253f9bc
    7f432657 5833833b 330a17b0 d04e9124 ad867d64 12dc744a 34a11d0a ea961d0b
    15fca34b 3bce6388 d0f82d0c 948610ca b69a3dca eb379c00 48358629 5078e845
    63cd1941 4ff595ec 7b98d4c4 71b350be 28b38fa0 b9539cf5 ca2c23a9 fd1406e8
    18b49ae8 3c6e81fd e4cd3536 b351d369 ec12ba56 6e6f9b57 c58b14e7 0ec79ced
    4a546ac9 4dc5bf11 b1ae1c67 81cb4455 33997f24 9b3f5345 7f861af3 3cfa6d7f
    81f5b84a d3f58537 1cb5a6d0 09e4187b 384efa0f 02030100 01a38201 df308201
    db303406 082b0601 05050701 01042830 26302406 082b0601 05050730 01861868
    7474703a 2f2f6f63 73702e76 65726973 69676e2e 636f6d30 12060355 1d130101
    ff040830 060101ff 02010030 70060355 1d200469 30673065 060b6086 480186f8
    45010717 03305630 2806082b 06010505 07020116 1c687474 70733a2f 2f777777
    2e766572 69736967 6e2e636f 6d2f6370 73302a06 082b0601 05050702 02301e1a
    1c687474 70733a2f 2f777777 2e766572 69736967 6e2e636f 6d2f7270 61303406
    03551d1f 042d302b 3029a027 a0258623 68747470 3a2f2f63 726c2e76 65726973
    <--- More --->
    69676e2e 636f6d2f 70636133 2d67352e 63726c30 0e060355 1d0f0101 ff040403
    02010630 6d06082b 06010505 07010c04 61305fa1 5da05b30 59305730 55160969
    6d616765 2f676966 3021301f 30070605 2b0e0302 1a04148f e5d31a86 ac8d8e6b
    c3cf806a d448182c 7b192e30 25162368 7474703a 2f2f6c6f 676f2e76 65726973
    69676e2e 636f6d2f 76736c6f 676f2e67 69663028 0603551d 11042130 1fa41d30
    1b311930 17060355 04031310 56657269 5369676e 4d504b49 2d322d36 301d0603
    551d0e04 1604140d 445c1653 44c1827e 1d20ab25 f40163d8 be79a530 1f060355
    1d230418 30168014 7fd365a7 c2ddecbb f03009f3 4339fa02 af333133 300d0609
    2a864886 f70d0101 05050003 82010100 0c8324ef ddc30cd9 589cfe36 b6eb8a80
    4bd1a3f7 9df3cc53 ef829ea3 a1e697c1 589d756c e01d1b4c fad1c12d 05c0ea6e
    b2227055 d9203340 3307c265 83fa8f43 379bea0e 9a6c70ee f69c803b d937f47a
    6decd018 7d494aca 99c71928 a2bed877 24f78526 866d8705 404167d1 273aeddc
    481d22cd 0b0b8bbc f4b17bfd b499a8e9 762ae11a 2d876e74 d388dd1e 22c6df16
    b62b8214 0a945cf2 50ecafce ff62370d ad65d306 4153ed02 14c8b558 28a1ace0
    5becb37f 954afb03 c8ad26db e6667812 4ad99f42 fbe198e6 42839b8f 8f6724e8
    6119b5dd cdb50b26 058ec36e c4c875b8 46cfe218 065ea9ae a8819a47 16de0c28
    6c2527b9 deb78458 c61f381e a4c4cb66
    quit
    telnet timeout 5
    ssh timeout 5
    console timeout 0
    dhcpd auto_config outside
    dhcpd address 172.16.100.5-172.16.101.4 inside
    <--- More --->
    threat-detection basic-threat
    threat-detection statistics access-list
    no threat-detection statistics tcp-intercept
    webvpn
    class-map inspection_default
    match default-inspection-traffic
    policy-map type inspect dns preset_dns_map
    parameters
    message-length maximum client auto
    message-length maximum 512
    policy-map global_policy
    class inspection_default
    inspect dns preset_dns_map
    inspect ftp
    inspect h323 h225
    inspect h323 ras
    inspect rsh
    inspect rtsp
    inspect esmtp
    <--- More --->
    inspect sqlnet
    inspect skinny
    inspect sunrpc
    inspect xdmcp
    inspect sip
    inspect netbios
    inspect tftp
    inspect ip-options
    inspect icmp
    inspect http
    service-policy global_policy global
    prompt hostname context
    call-home reporting anonymous
    Cryptochecksum:da1f2a6b2477754c30dfaef9172b8ed8
    : end

    Dear All
    thank you very much for your replies but nothing solved my problem
    Dear Ramraj
    when i am trying to do this command :no route inside 172.16.100.0 255.255.252.0 172.16.100.1 it gives me an ERROR
    (ERROR: Cannot remove connected route)
    could you all please help me
    thank you
    this is the newest running-config
    ASA Version 8.2(5)
    hostname ConcordeASA
    enable password 8Ry2YjIyt7RRXU24 encrypted
    passwd 2KFQnbNIdI.2KYOU encrypted
    names
    interface Ethernet0/0
    switchport access vlan 2
    interface Ethernet0/1
    interface Ethernet0/2
    interface Ethernet0/3
    interface Ethernet0/4
    switchport access vlan 12
    interface Ethernet0/5
    switchport access vlan 12
    interface Ethernet0/6
    <--- More --->
    <--- More --->
    interface Ethernet0/7
    <--- More --->
    interface Vlan1
    nameif inside
    security-level 100
    ip address 172.16.100.1 255.255.252.0
    interface Vlan2
    nameif outside
    security-level 0
    ip address 1.1.1.1   0.0.0.0
    ftp mode passive
    dns domain-lookup inside
    dns domain-lookup outside
    dns server-group DefaultDNS
    name-server 212.77.192.59
    name-server 212.77.192.60
    same-security-traffic permit intra-interface
    object-group protocol TCPUDP
    protocol-object udp
    protocol-object tcp
    access-list inside extended deny tcp host 172.16.100.197 any eq www
    access-list inside extended permit ip any any
    pager lines 24
    <--- More --->
    logging asdm informational
    mtu inside 1500
    mtu outside 1500
    icmp unreachable rate-limit 1 burst-size 1
    no asdm history enable
    arp timeout 14400
    global (outside) 1 interface
    nat (inside) 1 0.0.0.0 0.0.0.0
    access-group inside in interface inside
    route outside 0.0.0.0 0.0.0.0 1.1.1.1
    route inside 172.16.100.0 255.255.252.0 172.16.100.1 1
    timeout xlate 3:00:00
    timeout conn 1:00:00 half-closed 0:10:00 udp 0:02:00 icmp 0:00:02
    timeout sunrpc 0:10:00 h323 0:05:00 h225 1:00:00 mgcp 0:05:00 mgcp-pat 0:05:00
    timeout sip 0:30:00 sip_media 0:02:00 sip-invite 0:03:00 sip-disconnect 0:02:00
    timeout sip-provisional-media 0:02:00 uauth 0:05:00 absolute
    timeout tcp-proxy-reassembly 0:01:00
    timeout floating-conn 0:00:00
    dynamic-access-policy-record DfltAccessPolicy
    http server enable
    http 172.16.100.0 255.255.252.0 inside
    no snmp-server location
    no snmp-server contact
    snmp-server enable traps snmp authentication linkup linkdown coldstart
    <--- More --->
    crypto ipsec security-association lifetime seconds 28800
    crypto ipsec security-association lifetime kilobytes 4608000
    crypto ca trustpoint _SmartCallHome_ServerCA
    crl configure
    crypto ca certificate chain _SmartCallHome_ServerCA
    certificate ca 6ecc7aa5a7032009b8cebcf4e952d491
        308205ec 308204d4 a0030201 0202106e cc7aa5a7 032009b8 cebcf4e9 52d49130
        0d06092a 864886f7 0d010105 05003081 ca310b30 09060355 04061302 55533117
        30150603 55040a13 0e566572 69536967 6e2c2049 6e632e31 1f301d06 0355040b
        13165665 72695369 676e2054 72757374 204e6574 776f726b 313a3038 06035504
        0b133128 63292032 30303620 56657269 5369676e 2c20496e 632e202d 20466f72
        20617574 686f7269 7a656420 75736520 6f6e6c79 31453043 06035504 03133c56
        65726953 69676e20 436c6173 73203320 5075626c 69632050 72696d61 72792043
        65727469 66696361 74696f6e 20417574 686f7269 7479202d 20473530 1e170d31
        30303230 38303030 3030305a 170d3230 30323037 32333539 35395a30 81b5310b
        30090603 55040613 02555331 17301506 0355040a 130e5665 72695369 676e2c20
        496e632e 311f301d 06035504 0b131656 65726953 69676e20 54727573 74204e65
        74776f72 6b313b30 39060355 040b1332 5465726d 73206f66 20757365 20617420
        68747470 733a2f2f 7777772e 76657269 7369676e 2e636f6d 2f727061 20286329
        3130312f 302d0603 55040313 26566572 69536967 6e20436c 61737320 33205365
        63757265 20536572 76657220 4341202d 20473330 82012230 0d06092a 864886f7
        0d010101 05000382 010f0030 82010a02 82010100 b187841f c20c45f5 bcab2597
        a7ada23e 9cbaf6c1 39b88bca c2ac56c6 e5bb658e 444f4dce 6fed094a d4af4e10
        9c688b2e 957b899b 13cae234 34c1f35b f3497b62 83488174 d188786c 0253f9bc
    <--- More --->
        7f432657 5833833b 330a17b0 d04e9124 ad867d64 12dc744a 34a11d0a ea961d0b
        15fca34b 3bce6388 d0f82d0c 948610ca b69a3dca eb379c00 48358629 5078e845
        63cd1941 4ff595ec 7b98d4c4 71b350be 28b38fa0 b9539cf5 ca2c23a9 fd1406e8
        18b49ae8 3c6e81fd e4cd3536 b351d369 ec12ba56 6e6f9b57 c58b14e7 0ec79ced
        4a546ac9 4dc5bf11 b1ae1c67 81cb4455 33997f24 9b3f5345 7f861af3 3cfa6d7f
        81f5b84a d3f58537 1cb5a6d0 09e4187b 384efa0f 02030100 01a38201 df308201
        db303406 082b0601 05050701 01042830 26302406 082b0601 05050730 01861868
        7474703a 2f2f6f63 73702e76 65726973 69676e2e 636f6d30 12060355 1d130101
        ff040830 060101ff 02010030 70060355 1d200469 30673065 060b6086 480186f8
        45010717 03305630 2806082b 06010505 07020116 1c687474 70733a2f 2f777777
        2e766572 69736967 6e2e636f 6d2f6370 73302a06 082b0601 05050702 02301e1a
        1c687474 70733a2f 2f777777 2e766572 69736967 6e2e636f 6d2f7270 61303406
        03551d1f 042d302b 3029a027 a0258623 68747470 3a2f2f63 726c2e76 65726973
        69676e2e 636f6d2f 70636133 2d67352e 63726c30 0e060355 1d0f0101 ff040403
        02010630 6d06082b 06010505 07010c04 61305fa1 5da05b30 59305730 55160969
        6d616765 2f676966 3021301f 30070605 2b0e0302 1a04148f e5d31a86 ac8d8e6b
        c3cf806a d448182c 7b192e30 25162368 7474703a 2f2f6c6f 676f2e76 65726973
        69676e2e 636f6d2f 76736c6f 676f2e67 69663028 0603551d 11042130 1fa41d30
        1b311930 17060355 04031310 56657269 5369676e 4d504b49 2d322d36 301d0603
        551d0e04 1604140d 445c1653 44c1827e 1d20ab25 f40163d8 be79a530 1f060355
        1d230418 30168014 7fd365a7 c2ddecbb f03009f3 4339fa02 af333133 300d0609
        2a864886 f70d0101 05050003 82010100 0c8324ef ddc30cd9 589cfe36 b6eb8a80
        4bd1a3f7 9df3cc53 ef829ea3 a1e697c1 589d756c e01d1b4c fad1c12d 05c0ea6e
        b2227055 d9203340 3307c265 83fa8f43 379bea0e 9a6c70ee f69c803b d937f47a
    <--- More --->
        6decd018 7d494aca 99c71928 a2bed877 24f78526 866d8705 404167d1 273aeddc
        481d22cd 0b0b8bbc f4b17bfd b499a8e9 762ae11a 2d876e74 d388dd1e 22c6df16
        b62b8214 0a945cf2 50ecafce ff62370d ad65d306 4153ed02 14c8b558 28a1ace0
        5becb37f 954afb03 c8ad26db e6667812 4ad99f42 fbe198e6 42839b8f 8f6724e8
        6119b5dd cdb50b26 058ec36e c4c875b8 46cfe218 065ea9ae a8819a47 16de0c28
        6c2527b9 deb78458 c61f381e a4c4cb66
      quit
    telnet timeout 5
    ssh timeout 5
    console timeout 0
    dhcpd auto_config outside
    dhcpd address 172.16.100.5-172.16.101.4 inside
    priority-queue inside
    threat-detection basic-threat
    threat-detection statistics access-list
    no threat-detection statistics tcp-intercept
    webvpn
    class-map inspection_default
    match default-inspection-traffic
    <--- More --->
    policy-map type inspect dns preset_dns_map
    parameters
      message-length maximum client auto
      message-length maximum 512
    policy-map global_policy
    class inspection_default
      inspect dns preset_dns_map
      inspect ftp
      inspect h323 h225
      inspect h323 ras
      inspect rsh
      inspect rtsp
      inspect esmtp
      inspect sqlnet
      inspect skinny 
      inspect sunrpc
      inspect xdmcp
      inspect sip 
      inspect netbios
      inspect tftp
      inspect ip-options
      inspect icmp
      inspect http
    <--- More --->
    service-policy global_policy global
    prompt hostname context
    call-home reporting anonymous
    Cryptochecksum:bf1a120dc577a777b78b98d9ee887b04
    : end

  • Problem with nat-ing on asa 5505

    i have the asa5505 with asa8.4.2 and asdm 6.4.5. i use this asa5505 for connecting my network 192.168.0.0/24 with network 10.15.100.0/24. my wan port of asa5505 on network 10.13.74.0/24, lan port is on 192.168.0.0./24. this configuration worked ok until my isp changed router on address 10.13.74.1. i nat-ed on asa5505, i puted access policy and i had access network 10.15.100.0/24. but now i can't. the users from network can access devices on addresses 192.168.0.20 and 192.168.0.22 but i can't access the network 10.15.100.0/24. my configuration of asa5505 is:
    Result of the command: "show runn": Saved:ASA Version 8.4(2) !hostname ciscoasaenable password 8Ry2YjIyt7RRXU24 encryptedpasswd 2KFQnbNIdI.2KYOU encryptednames!interface Ethernet0/0 switchport access vlan 2!interface Ethernet0/1!interface Ethernet0/2!interface Ethernet0/3!interface Ethernet0/4!interface Ethernet0/5!interface Ethernet0/6!interface Ethernet0/7!interface Vlan1 nameif inside security-level 100 ip address 192.168.0.17 255.255.255.0 !interface Vlan2 nameif outside security-level 0 ip address 10.13.74.33 255.255.255.0 !ftp mode passiveobject network obj_any subnet 0.0.0.0 0.0.0.0object network server host 192.168.0.20object network sharepointdri host 192.168.0.22object network paragraflex host 192.168.0.20object network dri.local subnet 192.168.0.0 255.255.255.0object service ParagrafLex1 service tcp source eq 6190 description Odlazniobject service paragraf service tcp destination eq 6190 description dolazniobject network nonat host 192.168.0.20object network lokalnamreza range 192.168.0.1 192.168.0.254object network natnetwork subnet 192.168.0.0 255.255.255.0object network natmreze subnet 192.168.0.0 255.255.255.0object-group service DM_INLINE_SERVICE_2 service-object ip service-object icmp echo-reply service-object tcp object-group service DM_INLINE_SERVICE_1 service-object icmp echo-reply service-object tcp service-object ip service-object tcp destination eq domain service-object tcp destination eq ldap service-object object ParagrafLex1 object-group service DM_INLINE_SERVICE_8 service-object ip service-object tcp service-object icmp echo-replyobject-group service DM_INLINE_SERVICE_3 service-object tcp service-object tcp destination eq domain service-object tcp destination eq ldap object-group service DM_INLINE_SERVICE_4 service-object tcp service-object icmp echo-replyobject-group protocol DM_INLINE_PROTOCOL_2 protocol-object udp protocol-object tcpobject-group protocol TCPUDP protocol-object udp protocol-object tcpobject-group service DM_INLINE_SERVICE_5 service-object ip service-object icmp echo-replyobject-group protocol DM_INLINE_PROTOCOL_1 protocol-object ip protocol-object tcpobject-group service DM_INLINE_SERVICE_6 service-object ip service-object tcp service-object icmp echo-reply service-object icmp service-object tcp destination eq https object-group service DM_INLINE_SERVICE_7 service-object ip service-object tcp service-object icmp echo-reply service-object tcp destination eq https object-group network DM_INLINE_NETWORK_1 network-object 10.13.74.0 255.255.255.0 network-object 10.15.100.0 255.255.255.0object-group service DM_INLINE_SERVICE_9 service-object tcp-udp service-object tcp destination eq https service-object tcp destination eq domain object-group service DM_INLINE_SERVICE_10 service-object ip service-object tcp service-object icmp echo-replyobject-group service DM_INLINE_SERVICE_11 service-object ip service-object tcp service-object icmp echo-replyaccess-list nonat extended permit object-group DM_INLINE_SERVICE_8 192.168.0.0 255.255.255.0 object-group DM_INLINE_NETWORK_1 access-list inside_access_out extended permit object-group DM_INLINE_SERVICE_6 any any access-list inside_access_in extended permit object-group DM_INLINE_SERVICE_2 object dri.local 10.15.100.0 255.255.255.0 access-list inside_access_in extended permit object-group DM_INLINE_SERVICE_7 any any access-list uprava_access_out extended permit object-group DM_INLINE_SERVICE_3 object dri.local 10.13.74.0 255.255.255.0 access-list uprava_access_out extended permit object-group DM_INLINE_SERVICE_4 any any access-list uprava_access_out extended permit object-group DM_INLINE_SERVICE_3 192.168.0.0 255.255.255.0 10.13.74.0 255.255.255.0 access-list outside_access_in_1 extended permit object paragraf any object server access-list outside_access_in_1 extended permit object-group DM_INLINE_SERVICE_1 any object server access-list outside_access_in_1 extended permit object-group DM_INLINE_PROTOCOL_1 any object sharepointdri access-list outside_access_in_1 extended permit object-group DM_INLINE_SERVICE_10 object natmreze any access-list outside_access_out extended permit object-group DM_INLINE_SERVICE_9 any any access-list outside_access_out extended permit object-group DM_INLINE_SERVICE_11 object natmreze 10.15.100.0 255.255.255.0 pager lines 24logging asdm informationalmtu inside 1500mtu outside 1500icmp unreachable rate-limit 1 burst-size 1no asdm history enablearp outside 10.13.74.1 000d.bd64.a8e2 arp timeout 14400!object network server nat (inside,outside) static 10.13.74.34 dnsobject network sharepointdri nat (any,any) static 10.13.74.39object network nonat nat (inside,outside) static 192.168.0.20object network natmreze nat (any,any) static 10.13.74.42 dnsaccess-group inside_access_in in interface insideaccess-group inside_access_out out interface insideaccess-group outside_access_in_1 in interface outsideaccess-group outside_access_out out interface outsideroute outside 0.0.0.0 0.0.0.0 10.13.74.1 1route outside 10.15.100.0 255.255.255.0 10.13.74.1 1timeout xlate 3:00:00timeout conn 1:00:00 half-closed 0:10:00 udp 0:02:00 icmp 0:00:02timeout sunrpc 0:10:00 h323 0:05:00 h225 1:00:00 mgcp 0:05:00 mgcp-pat 0:05:00timeout sip 0:30:00 sip_media 0:02:00 sip-invite 0:03:00 sip-disconnect 0:02:00timeout sip-provisional-media 0:02:00 uauth 0:05:00 absolutetimeout tcp-proxy-reassembly 0:01:00timeout floating-conn 0:00:00dynamic-access-policy-record DfltAccessPolicyuser-identity default-domain LOCALhttp server enablehttp 192.168.0.0 255.255.255.0 insideno snmp-server locationno snmp-server contactsnmp-server enable traps snmp authentication linkup linkdown coldstart warmstarttelnet timeout 5ssh timeout 5console timeout 0dhcpd auto_config outside!threat-detection basic-threatthreat-detection statistics access-listno threat-detection statistics tcp-interceptwebvpn!class-map inspection_default match default-inspection-traffic!!policy-map type inspect dns preset_dns_map parameters  message-length maximum client auto  message-length maximum 512policy-map type inspect ftp paragraf parameterspolicy-map global_policy class inspection_default  inspect dns   inspect icmp   inspect ip-options   inspect netbios   inspect tftp   inspect h323 h225   inspect h323 ras !service-policy global_policy globalprompt hostname context state priority domain no call-home reporting anonymousCryptochecksum:61572938ed01b1c7447e43fcb2df4bc8: end
    what i do? plz help me?
    thanks

    Please do this, and let me know how it goes
    no access-list nonat extended permit object-group DM_INLINE_SERVICE_8 192.168.0.0 255.255.255.0 object-group DM_INLINE_NETWORK_1
    no access-list uprava_access_out extended permit object-group DM_INLINE_SERVICE_3 object dri.local 10.13.74.0 255.255.255.0
    no access-list uprava_access_out extended permit object-group DM_INLINE_SERVICE_4 any any
    no access-list uprava_access_out extended permit object-group DM_INLINE_SERVICE_3 192.168.0.0 255.255.255.0 10.13.74.0 255.255.255.0
    access-list inside_access_in line 1 permit ip 192.168.0.0 255.255.255.0 any
    access-list outside_access_in_1 line 1 permit ip any 192.168.0.0 255.255.255.0
    no object network nonat
    no access-group inside_access_out out interface inside
    no access-group outside_access_out out interface outside
    no route outside 10.15.100.0 255.255.255.0 10.13.74.1 1

  • Problem with Remote Access VPN on ASA 5505

    I am currently having an issue configuring an ASA 5505 to connect via remote access VPN using the Cisco VPN Client 5.0.07.0440 running on Windows 8 Pro x64. The VPN client prompts for the username and password during the connect process, but fails soon after.
    The VPN client logs are as follows:
    Cisco Systems VPN Client Version 5.0.07.0440
    Copyright (C) 1998-2010 Cisco Systems, Inc. All Rights Reserved.
    Client Type(s): Windows, WinNT
    Running on: 6.2.9200
    2      15:09:21.240  12/11/12  Sev=Info/4    CM/0x63100002
    Begin connection process
    3      15:09:21.287  12/11/12  Sev=Info/4    CM/0x63100004
    Establish secure connection
    4      15:09:21.287  12/11/12  Sev=Info/4    CM/0x63100024
    Attempt connection with server "**.**.***.***"
    5      15:09:21.287  12/11/12  Sev=Info/6    IKE/0x6300003B
    Attempting to establish a connection with **.**.***.***.
    6      15:09:21.287  12/11/12  Sev=Info/4    IKE/0x63000001
    Starting IKE Phase 1 Negotiation
    7      15:09:21.303  12/11/12  Sev=Info/4    IKE/0x63000013
    SENDING >>> ISAKMP OAK AG (SA, KE, NON, ID, VID(Xauth), VID(dpd), VID(Frag), VID(Nat-T), VID(Unity)) to **.**.***.***
    8      15:09:21.365  12/11/12  Sev=Info/6    GUI/0x63B00012
    Authentication request attributes is 6h.
    9      15:09:21.334  12/11/12  Sev=Info/5    IKE/0x6300002F
    Received ISAKMP packet: peer = **.**.***.***
    10     15:09:21.334  12/11/12  Sev=Info/4    IKE/0x63000014
    RECEIVING <<< ISAKMP OAK AG (SA, KE, NON, ID, HASH, VID(Unity), VID(Xauth), VID(dpd), VID(Nat-T), NAT-D, NAT-D, VID(Frag), VID(?)) from **.**.***.***
    11     15:09:21.334  12/11/12  Sev=Info/5    IKE/0x63000001
    Peer is a Cisco-Unity compliant peer
    12     15:09:21.334  12/11/12  Sev=Info/5    IKE/0x63000001
    Peer supports XAUTH
    13     15:09:21.334  12/11/12  Sev=Info/5    IKE/0x63000001
    Peer supports DPD
    14     15:09:21.334  12/11/12  Sev=Info/5    IKE/0x63000001
    Peer supports NAT-T
    15     15:09:21.334  12/11/12  Sev=Info/5    IKE/0x63000001
    Peer supports IKE fragmentation payloads
    16     15:09:21.334  12/11/12  Sev=Info/6    IKE/0x63000001
    IOS Vendor ID Contruction successful
    17     15:09:21.334  12/11/12  Sev=Info/4    IKE/0x63000013
    SENDING >>> ISAKMP OAK AG *(HASH, NOTIFY:STATUS_INITIAL_CONTACT, NAT-D, NAT-D, VID(?), VID(Unity)) to **.**.***.***
    18     15:09:21.334  12/11/12  Sev=Info/6    IKE/0x63000055
    Sent a keepalive on the IPSec SA
    19     15:09:21.334  12/11/12  Sev=Info/4    IKE/0x63000083
    IKE Port in use - Local Port =  0xFBCE, Remote Port = 0x1194
    20     15:09:21.334  12/11/12  Sev=Info/5    IKE/0x63000072
    Automatic NAT Detection Status:
       Remote end is NOT behind a NAT device
       This   end IS behind a NAT device
    21     15:09:21.334  12/11/12  Sev=Info/4    CM/0x6310000E
    Established Phase 1 SA.  1 Crypto Active IKE SA, 0 User Authenticated IKE SA in the system
    22     15:09:21.365  12/11/12  Sev=Info/5    IKE/0x6300002F
    Received ISAKMP packet: peer = **.**.***.***
    23     15:09:21.365  12/11/12  Sev=Info/4    IKE/0x63000014
    RECEIVING <<< ISAKMP OAK TRANS *(HASH, ATTR) from **.**.***.***
    24     15:09:21.365  12/11/12  Sev=Info/4    CM/0x63100015
    Launch xAuth application
    25     15:09:21.474  12/11/12  Sev=Info/4    IPSEC/0x63700008
    IPSec driver successfully started
    26     15:09:21.474  12/11/12  Sev=Info/4    IPSEC/0x63700014
    Deleted all keys
    27     15:09:27.319  12/11/12  Sev=Info/4    CM/0x63100017
    xAuth application returned
    28     15:09:27.319  12/11/12  Sev=Info/4    IKE/0x63000013
    SENDING >>> ISAKMP OAK TRANS *(HASH, ATTR) to **.**.***.***
    29     15:09:27.365  12/11/12  Sev=Info/5    IKE/0x6300002F
    Received ISAKMP packet: peer = **.**.***.***
    30     15:09:27.365  12/11/12  Sev=Info/4    IKE/0x63000014
    RECEIVING <<< ISAKMP OAK TRANS *(HASH, ATTR) from **.**.***.***
    31     15:09:27.365  12/11/12  Sev=Info/4    IKE/0x63000013
    SENDING >>> ISAKMP OAK TRANS *(HASH, ATTR) to **.**.***.***
    32     15:09:27.365  12/11/12  Sev=Info/4    CM/0x6310000E
    Established Phase 1 SA.  1 Crypto Active IKE SA, 1 User Authenticated IKE SA in the system
    33     15:09:27.365  12/11/12  Sev=Info/5    IKE/0x6300005E
    Client sending a firewall request to concentrator
    34     15:09:27.365  12/11/12  Sev=Info/4    IKE/0x63000013
    SENDING >>> ISAKMP OAK TRANS *(HASH, ATTR) to **.**.***.***
    35     15:09:27.397  12/11/12  Sev=Info/5    IKE/0x6300002F
    Received ISAKMP packet: peer = **.**.***.***
    36     15:09:27.397  12/11/12  Sev=Info/4    IKE/0x63000014
    RECEIVING <<< ISAKMP OAK TRANS *(HASH, ATTR) from **.**.***.***
    37     15:09:27.397  12/11/12  Sev=Info/5    IKE/0x63000010
    MODE_CFG_REPLY: Attribute = INTERNAL_IPV4_ADDRESS: , value = 192.168.2.70
    38     15:09:27.397  12/11/12  Sev=Info/5    IKE/0x63000010
    MODE_CFG_REPLY: Attribute = INTERNAL_IPV4_NETMASK: , value = 255.255.255.0
    39     15:09:27.397  12/11/12  Sev=Info/5    IKE/0x63000010
    MODE_CFG_REPLY: Attribute = INTERNAL_IPV4_DNS(1): , value = 192.168.2.1
    40     15:09:27.397  12/11/12  Sev=Info/5    IKE/0x63000010
    MODE_CFG_REPLY: Attribute = INTERNAL_IPV4_DNS(2): , value = 8.8.8.8
    41     15:09:27.397  12/11/12  Sev=Info/5    IKE/0x6300000D
    MODE_CFG_REPLY: Attribute = MODECFG_UNITY_SAVEPWD: , value = 0x00000001
    42     15:09:27.397  12/11/12  Sev=Info/5    IKE/0x6300000E
    MODE_CFG_REPLY: Attribute = MODECFG_UNITY_DEFDOMAIN: , value = NCHCO
    43     15:09:27.397  12/11/12  Sev=Info/5    IKE/0x6300000D
    MODE_CFG_REPLY: Attribute = MODECFG_UNITY_PFS: , value = 0x00000000
    44     15:09:27.397  12/11/12  Sev=Info/5    IKE/0x6300000E
    MODE_CFG_REPLY: Attribute = APPLICATION_VERSION, value = Cisco Systems, Inc ASA5505 Version 8.2(5) built by builders on Fri 20-May-11 16:00
    45     15:09:27.397  12/11/12  Sev=Info/5    IKE/0x6300000D
    MODE_CFG_REPLY: Attribute = MODECFG_UNITY_SMARTCARD_REMOVAL_DISCONNECT: , value = 0x00000001
    46     15:09:27.397  12/11/12  Sev=Info/5    IKE/0x6300000D
    MODE_CFG_REPLY: Attribute = Received and using NAT-T port number , value = 0x00001194
    47     15:09:27.397  12/11/12  Sev=Info/4    CM/0x63100019
    Mode Config data received
    48     15:09:27.412  12/11/12  Sev=Info/4    IKE/0x63000056
    Received a key request from Driver: Local IP = 192.168.2.70, GW IP = **.**.***.***, Remote IP = 0.0.0.0
    49     15:09:27.412  12/11/12  Sev=Info/4    IKE/0x63000013
    SENDING >>> ISAKMP OAK QM *(HASH, SA, NON, ID, ID) to **.**.***.***
    50     15:09:27.444  12/11/12  Sev=Info/5    IKE/0x6300002F
    Received ISAKMP packet: peer = **.**.***.***
    51     15:09:27.444  12/11/12  Sev=Info/4    IKE/0x63000014
    RECEIVING <<< ISAKMP OAK INFO *(HASH, NOTIFY:STATUS_RESP_LIFETIME) from **.**.***.***
    52     15:09:27.444  12/11/12  Sev=Info/5    IKE/0x63000045
    RESPONDER-LIFETIME notify has value of 86400 seconds
    53     15:09:27.444  12/11/12  Sev=Info/5    IKE/0x63000047
    This SA has already been alive for 6 seconds, setting expiry to 86394 seconds from now
    54     15:09:27.459  12/11/12  Sev=Info/5    IKE/0x6300002F
    Received ISAKMP packet: peer = **.**.***.***
    55     15:09:27.459  12/11/12  Sev=Info/4    IKE/0x63000014
    RECEIVING <<< ISAKMP OAK INFO *(HASH, NOTIFY:NO_PROPOSAL_CHOSEN) from **.**.***.***
    56     15:09:27.459  12/11/12  Sev=Info/4    IKE/0x63000013
    SENDING >>> ISAKMP OAK INFO *(HASH, DEL) to **.**.***.***
    57     15:09:27.459  12/11/12  Sev=Info/4    IKE/0x63000049
    Discarding IPsec SA negotiation, MsgID=CE99A8A8
    58     15:09:27.459  12/11/12  Sev=Info/4    IKE/0x63000017
    Marking IKE SA for deletion  (I_Cookie=A3A341F1C7606AD5 R_Cookie=F1F403018625E924) reason = DEL_REASON_IKE_NEG_FAILED
    59     15:09:27.459  12/11/12  Sev=Info/5    IKE/0x6300002F
    Received ISAKMP packet: peer = **.**.***.***
    60     15:09:27.459  12/11/12  Sev=Info/4    IKE/0x63000058
    Received an ISAKMP message for a non-active SA, I_Cookie=A3A341F1C7606AD5 R_Cookie=F1F403018625E924
    61     15:09:27.459  12/11/12  Sev=Info/4    IKE/0x63000014
    RECEIVING <<< ISAKMP OAK INFO *(Dropped) from **.**.***.***
    62     15:09:27.490  12/11/12  Sev=Info/4    IPSEC/0x63700014
    Deleted all keys
    63     15:09:30.475  12/11/12  Sev=Info/4    IKE/0x6300004B
    Discarding IKE SA negotiation (I_Cookie=A3A341F1C7606AD5 R_Cookie=F1F403018625E924) reason = DEL_REASON_IKE_NEG_FAILED
    64     15:09:30.475  12/11/12  Sev=Info/4    CM/0x63100012
    Phase 1 SA deleted before first Phase 2 SA is up cause by "DEL_REASON_IKE_NEG_FAILED".  0 Crypto Active IKE SA, 0 User Authenticated IKE SA in the system
    65     15:09:30.475  12/11/12  Sev=Info/5    CM/0x63100025
    Initializing CVPNDrv
    66     15:09:30.475  12/11/12  Sev=Info/6    CM/0x63100046
    Set tunnel established flag in registry to 0.
    67     15:09:30.475  12/11/12  Sev=Info/4    IKE/0x63000001
    IKE received signal to terminate VPN connection
    68     15:09:30.475  12/11/12  Sev=Info/4    IPSEC/0x63700014
    Deleted all keys
    69     15:09:30.475  12/11/12  Sev=Info/4    IPSEC/0x63700014
    Deleted all keys
    70     15:09:30.475  12/11/12  Sev=Info/4    IPSEC/0x63700014
    Deleted all keys
    71     15:09:30.475  12/11/12  Sev=Info/4    IPSEC/0x6370000A
    IPSec driver successfully stopped
    The running configuration is as follows (there is a site-to-site VPN set up as well to another ASA 5505, but that is working flawlessly):
    : Saved
    ASA Version 8.2(5)
    hostname NCHCO
    enable password hTjwXz/V8EuTw9p9 encrypted
    passwd hTjwXz/V8EuTw9p9 encrypted
    names
    name 192.168.2.0 NCHCO description City Offices
    name 192.168.2.80 VPN_End
    name 192.168.2.70 VPN_Start
    interface Ethernet0/0
    switchport access vlan 2
    speed 100
    duplex full
    interface Ethernet0/1
    interface Ethernet0/2
    interface Ethernet0/3
    interface Ethernet0/4
    interface Ethernet0/5
    interface Ethernet0/6
    interface Ethernet0/7
    interface Vlan1
    nameif inside
    security-level 100
    ip address 192.168.2.1 255.255.255.0
    interface Vlan2
    nameif outside
    security-level 0
    ip address **.**.***.*** 255.255.255.248
    boot system disk0:/asa825-k8.bin
    ftp mode passive
    access-list outside_nat0_outbound extended permit ip NCHCO 255.255.255.0 192.168.1.0 255.255.255.0
    access-list inside_nat0_outbound extended permit ip NCHCO 255.255.255.0 192.168.1.0 255.255.255.0
    access-list inside_nat0_outbound extended permit ip any 192.168.2.64 255.255.255.224
    access-list outside_1_cryptomap extended permit ip NCHCO 255.255.255.0 192.168.1.0 255.255.255.0
    access-list outside_1_cryptomap_1 extended permit ip NCHCO 255.255.255.0 192.168.1.0 255.255.255.0
    access-list LAN_Access standard permit NCHCO 255.255.255.0
    access-list LAN_Access standard permit 0.0.0.0 255.255.255.0
    pager lines 24
    logging enable
    logging asdm informational
    mtu inside 1500
    mtu outside 1500
    ip local pool VPN_Pool VPN_Start-VPN_End mask 255.255.255.0
    icmp unreachable rate-limit 1 burst-size 1
    asdm image disk0:/asdm-645.bin
    no asdm history enable
    arp timeout 14400
    global (outside) 1 interface
    nat (inside) 0 access-list inside_nat0_outbound
    nat (inside) 1 0.0.0.0 0.0.0.0
    nat (outside) 0 access-list outside_nat0_outbound
    route outside 0.0.0.0 0.0.0.0 74.219.208.49 1
    timeout xlate 3:00:00
    timeout conn 1:00:00 half-closed 0:10:00 udp 0:02:00 icmp 0:00:02
    timeout sunrpc 0:10:00 h323 0:05:00 h225 1:00:00 mgcp 0:05:00 mgcp-pat 0:05:00
    timeout sip 0:30:00 sip_media 0:02:00 sip-invite 0:03:00 sip-disconnect 0:02:00
    timeout sip-provisional-media 0:02:00 uauth 0:05:00 absolute
    timeout tcp-proxy-reassembly 0:01:00
    timeout floating-conn 0:00:00
    dynamic-access-policy-record DfltAccessPolicy
    network-acl outside_nat0_outbound
    webvpn
      svc ask enable default svc
    http server enable
    http 192.168.1.0 255.255.255.0 inside
    http **.**.***.*** 255.255.255.255 outside
    http 74.218.158.238 255.255.255.255 outside
    http NCHCO 255.255.255.0 inside
    no snmp-server location
    no snmp-server contact
    snmp-server enable traps snmp authentication linkup linkdown coldstart
    crypto ipsec transform-set ESP-AES-128-SHA esp-aes esp-sha-hmac
    crypto ipsec transform-set ESP-AES-128-MD5 esp-aes esp-md5-hmac
    crypto ipsec transform-set ESP-AES-192-SHA esp-aes-192 esp-sha-hmac
    crypto ipsec transform-set ESP-AES-192-MD5 esp-aes-192 esp-md5-hmac
    crypto ipsec transform-set ESP-AES-256-SHA esp-aes-256 esp-sha-hmac
    crypto ipsec transform-set ESP-AES-256-MD5 esp-aes-256 esp-md5-hmac
    crypto ipsec transform-set ESP-3DES-MD5 esp-3des esp-md5-hmac
    crypto ipsec transform-set ESP-DES-SHA esp-des esp-sha-hmac
    crypto ipsec transform-set ESP-DES-MD5 esp-des esp-md5-hmac
    crypto ipsec transform-set l2tp-transform esp-3des esp-sha-hmac
    crypto ipsec transform-set l2tp-transform mode transport
    crypto ipsec transform-set vpn-transform esp-aes-256 esp-sha-hmac
    crypto ipsec transform-set TRANS_ESP_3DES_SHA esp-3des esp-sha-hmac
    crypto ipsec transform-set TRANS_ESP_3DES_SHA mode transport
    crypto ipsec transform-set TRANS_ESP_3DES_MD5 esp-3des esp-md5-hmac
    crypto ipsec transform-set TRANS_ESP_3DES_MD5 mode transport
    crypto ipsec transform-set ESP-3DES-SHA esp-3des esp-sha-hmac
    crypto ipsec security-association lifetime seconds 28800
    crypto ipsec security-association lifetime kilobytes 4608000
    crypto dynamic-map SYSTEM_DEFAULT_CRYPTO_MAP 65535 set pfs group1
    crypto dynamic-map SYSTEM_DEFAULT_CRYPTO_MAP 65535 set transform-set ESP-AES-128-SHA ESP-AES-128-MD5 ESP-AES-192-SHA ESP-AES-192-MD5 ESP-AES-256-SHA ESP-AES-256-MD5 ESP-3DES-SHA ESP-3DES-MD5 ESP-DES-SHA ESP-DES-MD5
    crypto dynamic-map dyn-map 10 set pfs group1
    crypto dynamic-map dyn-map 10 set transform-set l2tp-transform vpn-transform
    crypto dynamic-map dyn-map 10 set reverse-route
    crypto dynamic-map outside_dyn_map 20 set transform-set TRANS_ESP_3DES_MD5
    crypto map outside_map 1 match address outside_1_cryptomap
    crypto map outside_map 1 set pfs group1
    crypto map outside_map 1 set peer 74.219.208.50
    crypto map outside_map 1 set transform-set ESP-3DES-SHA
    crypto map outside_map 20 ipsec-isakmp dynamic outside_dyn_map
    crypto map outside_map 65535 ipsec-isakmp dynamic SYSTEM_DEFAULT_CRYPTO_MAP
    crypto map outside_map interface outside
    crypto map inside_map 65535 ipsec-isakmp dynamic SYSTEM_DEFAULT_CRYPTO_MAP
    crypto map inside_map interface inside
    crypto map vpn-map 1 match address outside_1_cryptomap_1
    crypto map vpn-map 1 set pfs group1
    crypto map vpn-map 1 set peer 74.219.208.50
    crypto map vpn-map 1 set transform-set ESP-3DES-SHA
    crypto map vpn-map 10 ipsec-isakmp dynamic dyn-map
    crypto isakmp identity address
    crypto isakmp enable inside
    crypto isakmp enable outside
    crypto isakmp policy 10
    authentication pre-share
    encryption 3des
    hash md5
    group 2
    lifetime 86400
    crypto isakmp policy 15
    authentication pre-share
    encryption aes-256
    hash sha
    group 2
    lifetime 86400
    crypto isakmp policy 35
    authentication pre-share
    encryption 3des
    hash sha
    group 2
    lifetime 86400
    crypto isakmp ipsec-over-tcp port 10000
    client-update enable
    telnet 192.168.1.0 255.255.255.0 inside
    telnet NCHCO 255.255.255.0 inside
    telnet timeout 5
    ssh 192.168.1.0 255.255.255.0 inside
    ssh NCHCO 255.255.255.0 inside
    ssh timeout 5
    console timeout 0
    dhcpd address 192.168.2.150-192.168.2.225 inside
    dhcpd dns 216.68.4.10 216.68.5.10 interface inside
    dhcpd lease 64000 interface inside
    threat-detection basic-threat
    threat-detection statistics access-list
    no threat-detection statistics tcp-intercept
    webvpn
    group-policy DefaultRAGroup internal
    group-policy DefaultRAGroup attributes
    dns-server value 192.168.2.1
    vpn-tunnel-protocol IPSec l2tp-ipsec
    default-domain value nchco.local
    group-policy DfltGrpPolicy attributes
    dns-server value 192.168.2.1
    vpn-tunnel-protocol IPSec l2tp-ipsec svc webvpn
    password-storage enable
    ipsec-udp enable
    intercept-dhcp 255.255.255.0 enable
    address-pools value VPN_Pool
    group-policy NCHVPN internal
    group-policy NCHVPN attributes
    dns-server value 192.168.2.1 8.8.8.8
    vpn-tunnel-protocol IPSec l2tp-ipsec
    default-domain value NCHCO
    username admin password LbMiJuAJjDaFb2uw encrypted privilege 15
    username 8njferg password yB1lHEVmHZGj5C2Z encrypted privilege 15
    username NCHvpn99 password QhZZtJfwbnowceB7 encrypted
    tunnel-group DefaultRAGroup general-attributes
    address-pool (inside) VPN_Pool
    address-pool VPN_Pool
    authentication-server-group (inside) LOCAL
    authentication-server-group (outside) LOCAL
    authorization-server-group LOCAL
    authorization-server-group (inside) LOCAL
    authorization-server-group (outside) LOCAL
    default-group-policy DefaultRAGroup
    strip-realm
    strip-group
    tunnel-group DefaultRAGroup ipsec-attributes
    pre-shared-key *****
    peer-id-validate nocheck
    tunnel-group DefaultRAGroup ppp-attributes
    no authentication chap
    no authentication ms-chap-v1
    authentication ms-chap-v2
    tunnel-group DefaultWEBVPNGroup ppp-attributes
    authentication pap
    authentication ms-chap-v2
    tunnel-group 74.219.208.50 type ipsec-l2l
    tunnel-group 74.219.208.50 ipsec-attributes
    pre-shared-key *****
    tunnel-group NCHVPN type remote-access
    tunnel-group NCHVPN general-attributes
    address-pool VPN_Pool
    default-group-policy NCHVPN
    tunnel-group NCHVPN ipsec-attributes
    pre-shared-key *****
    class-map inspection_default
    match default-inspection-traffic
    policy-map type inspect dns preset_dns_map
    parameters
      message-length maximum client auto
      message-length maximum 512
    policy-map global_policy
    class inspection_default
      inspect dns preset_dns_map
      inspect ftp
      inspect h323 h225
      inspect h323 ras
      inspect rsh
      inspect rtsp
      inspect esmtp
      inspect sqlnet
      inspect skinny
      inspect sunrpc
      inspect xdmcp
      inspect sip
      inspect netbios
      inspect tftp
      inspect ip-options
    service-policy global_policy global
    prompt hostname context
    no call-home reporting anonymous
    Cryptochecksum:15852745977ff159ba808c4a4feb61fa
    : end
    asdm image disk0:/asdm-645.bin
    asdm location VPN_Start 255.255.255.255 inside
    asdm location VPN_End 255.255.255.255 inside
    no asdm history enable
    Anyone have any idea why this is happening?
    Thanks!

    Thanks again for your reply, and sorry about the late response, havent gotten back to this issue until just now. I applied the above command as you specified, and unfortunately, it did not resolve the problem. Below are the logs from the VPN Client for the connection + attempted browsing of a network share that is behind the ASA, and the new running configuration.
    VPN Client Log:
    Cisco Systems VPN Client Version 5.0.07.0440
    Copyright (C) 1998-2010 Cisco Systems, Inc. All Rights Reserved.
    Client Type(s): Windows, WinNT
    Running on: 6.2.9200
    331    13:11:41.362  12/17/12  Sev=Info/4    CM/0x63100002
    Begin connection process
    332    13:11:41.362  12/17/12  Sev=Info/4    CM/0x63100004
    Establish secure connection
    333    13:11:41.362  12/17/12  Sev=Info/4    CM/0x63100024
    Attempt connection with server "69.61.228.178"
    334    13:11:41.362  12/17/12  Sev=Info/6    IKE/0x6300003B
    Attempting to establish a connection with 69.61.228.178.
    335    13:11:41.362  12/17/12  Sev=Info/4    IKE/0x63000001
    Starting IKE Phase 1 Negotiation
    336    13:11:41.424  12/17/12  Sev=Info/6    GUI/0x63B00012
    Authentication request attributes is 6h.
    337    13:11:41.362  12/17/12  Sev=Info/4    IKE/0x63000013
    SENDING >>> ISAKMP OAK AG (SA, KE, NON, ID, VID(Xauth), VID(dpd), VID(Frag), VID(Nat-T), VID(Unity)) to 69.61.228.178
    338    13:11:41.393  12/17/12  Sev=Info/5    IKE/0x6300002F
    Received ISAKMP packet: peer = 69.61.228.178
    339    13:11:41.393  12/17/12  Sev=Info/4    IKE/0x63000014
    RECEIVING <<< ISAKMP OAK AG (SA, KE, NON, ID, HASH, VID(Unity), VID(Xauth), VID(dpd), VID(Nat-T), NAT-D, NAT-D, VID(Frag), VID(?)) from 69.61.228.178
    340    13:11:41.393  12/17/12  Sev=Info/5    IKE/0x63000001
    Peer is a Cisco-Unity compliant peer
    341    13:11:41.393  12/17/12  Sev=Info/5    IKE/0x63000001
    Peer supports XAUTH
    342    13:11:41.393  12/17/12  Sev=Info/5    IKE/0x63000001
    Peer supports DPD
    343    13:11:41.393  12/17/12  Sev=Info/5    IKE/0x63000001
    Peer supports NAT-T
    344    13:11:41.393  12/17/12  Sev=Info/5    IKE/0x63000001
    Peer supports IKE fragmentation payloads
    345    13:11:41.393  12/17/12  Sev=Info/6    IKE/0x63000001
    IOS Vendor ID Contruction successful
    346    13:11:41.393  12/17/12  Sev=Info/4    IKE/0x63000013
    SENDING >>> ISAKMP OAK AG *(HASH, NOTIFY:STATUS_INITIAL_CONTACT, NAT-D, NAT-D, VID(?), VID(Unity)) to 69.61.228.178
    347    13:11:41.393  12/17/12  Sev=Info/6    IKE/0x63000055
    Sent a keepalive on the IPSec SA
    348    13:11:41.393  12/17/12  Sev=Info/4    IKE/0x63000083
    IKE Port in use - Local Port =  0xD271, Remote Port = 0x1194
    349    13:11:41.393  12/17/12  Sev=Info/5    IKE/0x63000072
    Automatic NAT Detection Status:
       Remote end is NOT behind a NAT device
       This   end IS behind a NAT device
    350    13:11:41.393  12/17/12  Sev=Info/4    CM/0x6310000E
    Established Phase 1 SA.  1 Crypto Active IKE SA, 0 User Authenticated IKE SA in the system
    351    13:11:41.424  12/17/12  Sev=Info/5    IKE/0x6300002F
    Received ISAKMP packet: peer = 69.61.228.178
    352    13:11:41.424  12/17/12  Sev=Info/4    IKE/0x63000014
    RECEIVING <<< ISAKMP OAK TRANS *(HASH, ATTR) from 69.61.228.178
    353    13:11:41.424  12/17/12  Sev=Info/4    CM/0x63100015
    Launch xAuth application
    354    13:11:41.424  12/17/12  Sev=Info/4    CM/0x63100017
    xAuth application returned
    355    13:11:41.424  12/17/12  Sev=Info/4    IKE/0x63000013
    SENDING >>> ISAKMP OAK TRANS *(HASH, ATTR) to 69.61.228.178
    356    13:11:41.456  12/17/12  Sev=Info/5    IKE/0x6300002F
    Received ISAKMP packet: peer = 69.61.228.178
    357    13:11:41.456  12/17/12  Sev=Info/4    IKE/0x63000014
    RECEIVING <<< ISAKMP OAK TRANS *(HASH, ATTR) from 69.61.228.178
    358    13:11:41.456  12/17/12  Sev=Info/4    IKE/0x63000013
    SENDING >>> ISAKMP OAK TRANS *(HASH, ATTR) to 69.61.228.178
    359    13:11:41.456  12/17/12  Sev=Info/4    CM/0x6310000E
    Established Phase 1 SA.  1 Crypto Active IKE SA, 1 User Authenticated IKE SA in the system
    360    13:11:41.456  12/17/12  Sev=Info/5    IKE/0x6300005E
    Client sending a firewall request to concentrator
    361    13:11:41.456  12/17/12  Sev=Info/4    IKE/0x63000013
    SENDING >>> ISAKMP OAK TRANS *(HASH, ATTR) to 69.61.228.178
    362    13:11:41.502  12/17/12  Sev=Info/5    IKE/0x6300002F
    Received ISAKMP packet: peer = 69.61.228.178
    363    13:11:41.502  12/17/12  Sev=Info/4    IKE/0x63000014
    RECEIVING <<< ISAKMP OAK TRANS *(HASH, ATTR) from 69.61.228.178
    364    13:11:41.502  12/17/12  Sev=Info/5    IKE/0x63000010
    MODE_CFG_REPLY: Attribute = INTERNAL_IPV4_ADDRESS: , value = 192.168.2.70
    365    13:11:41.502  12/17/12  Sev=Info/5    IKE/0x63000010
    MODE_CFG_REPLY: Attribute = INTERNAL_IPV4_NETMASK: , value = 255.255.255.0
    366    13:11:41.502  12/17/12  Sev=Info/5    IKE/0x63000010
    MODE_CFG_REPLY: Attribute = INTERNAL_IPV4_DNS(1): , value = 192.168.2.1
    367    13:11:41.502  12/17/12  Sev=Info/5    IKE/0x63000010
    MODE_CFG_REPLY: Attribute = INTERNAL_IPV4_DNS(2): , value = 8.8.8.8
    368    13:11:41.502  12/17/12  Sev=Info/5    IKE/0x6300000D
    MODE_CFG_REPLY: Attribute = MODECFG_UNITY_SAVEPWD: , value = 0x00000001
    369    13:11:41.502  12/17/12  Sev=Info/5    IKE/0x6300000D
    MODE_CFG_REPLY: Attribute = MODECFG_UNITY_SPLIT_INCLUDE (# of split_nets), value = 0x00000001
    370    13:11:41.502  12/17/12  Sev=Info/5    IKE/0x6300000F
    SPLIT_NET #1
        subnet = 192.168.2.0
        mask = 255.255.255.0
        protocol = 0
        src port = 0
        dest port=0
    371    13:11:41.502  12/17/12  Sev=Info/5    IKE/0x6300000E
    MODE_CFG_REPLY: Attribute = MODECFG_UNITY_DEFDOMAIN: , value = NCHCO.local
    372    13:11:41.502  12/17/12  Sev=Info/5    IKE/0x6300000D
    MODE_CFG_REPLY: Attribute = MODECFG_UNITY_PFS: , value = 0x00000000
    373    13:11:41.502  12/17/12  Sev=Info/5    IKE/0x6300000E
    MODE_CFG_REPLY: Attribute = APPLICATION_VERSION, value = Cisco Systems, Inc ASA5505 Version 8.4(1) built by builders on Mon 31-Jan-11 02:11
    374    13:11:41.502  12/17/12  Sev=Info/5    IKE/0x6300000D
    MODE_CFG_REPLY: Attribute = MODECFG_UNITY_SMARTCARD_REMOVAL_DISCONNECT: , value = 0x00000001
    375    13:11:41.502  12/17/12  Sev=Info/5    IKE/0x6300000D
    MODE_CFG_REPLY: Attribute = Received and using NAT-T port number , value = 0x00001194
    376    13:11:41.502  12/17/12  Sev=Info/4    CM/0x63100019
    Mode Config data received
    377    13:11:41.502  12/17/12  Sev=Info/4    IKE/0x63000056
    Received a key request from Driver: Local IP = 192.168.2.70, GW IP = 69.61.228.178, Remote IP = 0.0.0.0
    378    13:11:41.502  12/17/12  Sev=Info/4    IKE/0x63000013
    SENDING >>> ISAKMP OAK QM *(HASH, SA, NON, ID, ID) to 69.61.228.178
    379    13:11:41.534  12/17/12  Sev=Info/5    IKE/0x6300002F
    Received ISAKMP packet: peer = 69.61.228.178
    380    13:11:41.534  12/17/12  Sev=Info/4    IKE/0x63000014
    RECEIVING <<< ISAKMP OAK INFO *(HASH, NOTIFY:STATUS_RESP_LIFETIME) from 69.61.228.178
    381    13:11:41.534  12/17/12  Sev=Info/5    IKE/0x63000045
    RESPONDER-LIFETIME notify has value of 86400 seconds
    382    13:11:41.534  12/17/12  Sev=Info/5    IKE/0x63000047
    This SA has already been alive for 0 seconds, setting expiry to 86400 seconds from now
    383    13:11:41.549  12/17/12  Sev=Info/5    IKE/0x6300002F
    Received ISAKMP packet: peer = 69.61.228.178
    384    13:11:41.549  12/17/12  Sev=Info/4    IKE/0x63000014
    RECEIVING <<< ISAKMP OAK QM *(HASH, SA, NON, ID, ID, NOTIFY:STATUS_RESP_LIFETIME) from 69.61.228.178
    385    13:11:41.549  12/17/12  Sev=Info/5    IKE/0x63000045
    RESPONDER-LIFETIME notify has value of 28800 seconds
    386    13:11:41.549  12/17/12  Sev=Info/4    IKE/0x63000013
    SENDING >>> ISAKMP OAK QM *(HASH) to 69.61.228.178
    387    13:11:41.549  12/17/12  Sev=Info/5    IKE/0x63000059
    Loading IPsec SA (MsgID=C4F5B5A6 OUTBOUND SPI = 0xD2DBADEA INBOUND SPI = 0x14762837)
    388    13:11:41.549  12/17/12  Sev=Info/5    IKE/0x63000025
    Loaded OUTBOUND ESP SPI: 0xD2DBADEA
    389    13:11:41.549  12/17/12  Sev=Info/5    IKE/0x63000026
    Loaded INBOUND ESP SPI: 0x14762837
    390    13:11:41.549  12/17/12  Sev=Info/5    CVPND/0x63400013
        Destination           Netmask           Gateway         Interface   Metric
            0.0.0.0           0.0.0.0       192.168.1.1     192.168.1.162       10
          127.0.0.0         255.0.0.0         127.0.0.1         127.0.0.1      306
          127.0.0.1   255.255.255.255         127.0.0.1         127.0.0.1      306
    127.255.255.255   255.255.255.255         127.0.0.1         127.0.0.1      306
        192.168.1.0     255.255.255.0     192.168.1.162     192.168.1.162      266
      192.168.1.162   255.255.255.255     192.168.1.162     192.168.1.162      266
      192.168.1.255   255.255.255.255     192.168.1.162     192.168.1.162      266
          224.0.0.0         240.0.0.0         127.0.0.1         127.0.0.1      306
          224.0.0.0         240.0.0.0     192.168.1.162     192.168.1.162      266
    255.255.255.255   255.255.255.255         127.0.0.1         127.0.0.1      306
    255.255.255.255   255.255.255.255     192.168.1.162     192.168.1.162      266
    391    13:11:41.877  12/17/12  Sev=Info/6    CVPND/0x63400001
    Launch VAInst64 to control IPSec Virtual Adapter
    392    13:11:43.455  12/17/12  Sev=Info/4    CM/0x63100034
    The Virtual Adapter was enabled:
        IP=192.168.2.70/255.255.255.0
        DNS=192.168.2.1,8.8.8.8
        WINS=0.0.0.0,0.0.0.0
        Domain=NCHCO.local
        Split DNS Names=
    393    13:11:43.455  12/17/12  Sev=Info/5    CVPND/0x63400013
        Destination           Netmask           Gateway         Interface   Metric
            0.0.0.0           0.0.0.0       192.168.1.1     192.168.1.162       10
          127.0.0.0         255.0.0.0         127.0.0.1         127.0.0.1      306
          127.0.0.1   255.255.255.255         127.0.0.1         127.0.0.1      306
    127.255.255.255   255.255.255.255         127.0.0.1         127.0.0.1      306
        192.168.1.0     255.255.255.0     192.168.1.162     192.168.1.162      266
      192.168.1.162   255.255.255.255     192.168.1.162     192.168.1.162      266
      192.168.1.255   255.255.255.255     192.168.1.162     192.168.1.162      266
          224.0.0.0         240.0.0.0         127.0.0.1         127.0.0.1      306
          224.0.0.0         240.0.0.0     192.168.1.162     192.168.1.162      266
          224.0.0.0         240.0.0.0           0.0.0.0           0.0.0.0      266
    255.255.255.255   255.255.255.255         127.0.0.1         127.0.0.1      306
    255.255.255.255   255.255.255.255     192.168.1.162     192.168.1.162      266
    255.255.255.255   255.255.255.255           0.0.0.0           0.0.0.0      266
    394    13:11:47.517  12/17/12  Sev=Info/4    CM/0x63100038
    Successfully saved route changes to file.
    395    13:11:47.517  12/17/12  Sev=Info/5    CVPND/0x63400013
        Destination           Netmask           Gateway         Interface   Metric
            0.0.0.0           0.0.0.0       192.168.1.1     192.168.1.162       10
      69.61.228.178   255.255.255.255       192.168.1.1     192.168.1.162      100
          127.0.0.0         255.0.0.0         127.0.0.1         127.0.0.1      306
          127.0.0.1   255.255.255.255         127.0.0.1         127.0.0.1      306
    127.255.255.255   255.255.255.255         127.0.0.1         127.0.0.1      306
        192.168.1.0     255.255.255.0     192.168.1.162     192.168.1.162      266
        192.168.1.2   255.255.255.255     192.168.1.162     192.168.1.162      100
      192.168.1.162   255.255.255.255     192.168.1.162     192.168.1.162      266
      192.168.1.255   255.255.255.255     192.168.1.162     192.168.1.162      266
        192.168.2.0     255.255.255.0      192.168.2.70      192.168.2.70      266
        192.168.2.0     255.255.255.0       192.168.2.1      192.168.2.70      100
       192.168.2.70   255.255.255.255      192.168.2.70      192.168.2.70      266
      192.168.2.255   255.255.255.255      192.168.2.70      192.168.2.70      266
          224.0.0.0         240.0.0.0         127.0.0.1         127.0.0.1      306
          224.0.0.0         240.0.0.0     192.168.1.162     192.168.1.162      266
          224.0.0.0         240.0.0.0      192.168.2.70      192.168.2.70      266
    255.255.255.255   255.255.255.255         127.0.0.1         127.0.0.1      306
    255.255.255.255   255.255.255.255     192.168.1.162     192.168.1.162      266
    255.255.255.255   255.255.255.255      192.168.2.70      192.168.2.70      266
    396    13:11:47.517  12/17/12  Sev=Info/6    CM/0x63100036
    The routing table was updated for the Virtual Adapter
    397    13:11:47.517  12/17/12  Sev=Info/4    CM/0x6310001A
    One secure connection established
    398    13:11:47.517  12/17/12  Sev=Info/4    CM/0x6310003B
    Address watch added for 192.168.1.162.  Current hostname: MATT-PC, Current address(es): 192.168.2.70, 192.168.1.162.
    399    13:11:47.517  12/17/12  Sev=Info/4    CM/0x6310003B
    Address watch added for 192.168.2.70.  Current hostname: MATT-PC, Current address(es): 192.168.2.70, 192.168.1.162.
    400    13:11:47.517  12/17/12  Sev=Info/5    CM/0x63100001
    Did not find the Smartcard to watch for removal
    401    13:11:47.517  12/17/12  Sev=Info/4    IPSEC/0x63700008
    IPSec driver successfully started
    402    13:11:47.517  12/17/12  Sev=Info/4    IPSEC/0x63700014
    Deleted all keys
    403    13:11:47.517  12/17/12  Sev=Info/6    IPSEC/0x6370002C
    Sent 109 packets, 0 were fragmented.
    404    13:11:47.517  12/17/12  Sev=Info/4    IPSEC/0x63700014
    Deleted all keys
    405    13:11:47.517  12/17/12  Sev=Info/4    IPSEC/0x63700010
    Created a new key structure
    406    13:11:47.517  12/17/12  Sev=Info/4    IPSEC/0x6370000F
    Added key with SPI=0xeaaddbd2 into key list
    407    13:11:47.517  12/17/12  Sev=Info/4    IPSEC/0x63700010
    Created a new key structure
    408    13:11:47.517  12/17/12  Sev=Info/4    IPSEC/0x6370000F
    Added key with SPI=0x37287614 into key list
    409    13:11:47.517  12/17/12  Sev=Info/4    IPSEC/0x6370002F
    Assigned VA private interface addr 192.168.2.70
    410    13:11:47.517  12/17/12  Sev=Info/4    IPSEC/0x63700037
    Configure public interface: 192.168.1.162. SG: 69.61.228.178
    411    13:11:47.517  12/17/12  Sev=Info/6    CM/0x63100046
    Set tunnel established flag in registry to 1.
    412    13:11:52.688  12/17/12  Sev=Info/4    IKE/0x63000013
    SENDING >>> ISAKMP OAK INFO *(HASH, NOTIFY:DPD_REQUEST) to 69.61.228.178
    413    13:11:52.688  12/17/12  Sev=Info/6    IKE/0x6300003D
    Sending DPD request to 69.61.228.178, our seq# = 2722476009
    414    13:11:52.704  12/17/12  Sev=Info/5    IKE/0x6300002F
    Received ISAKMP packet: peer = 69.61.228.178
    415    13:11:52.704  12/17/12  Sev=Info/4    IKE/0x63000014
    RECEIVING <<< ISAKMP OAK INFO *(HASH, NOTIFY:DPD_ACK) from 69.61.228.178
    416    13:11:52.704  12/17/12  Sev=Info/5    IKE/0x63000040
    Received DPD ACK from 69.61.228.178, seq# received = 2722476009, seq# expected = 2722476009
    417    13:12:03.187  12/17/12  Sev=Info/4    IKE/0x63000013
    SENDING >>> ISAKMP OAK INFO *(HASH, NOTIFY:DPD_REQUEST) to 69.61.228.178
    418    13:12:03.187  12/17/12  Sev=Info/6    IKE/0x6300003D
    Sending DPD request to 69.61.228.178, our seq# = 2722476010
    419    13:12:03.202  12/17/12  Sev=Info/5    IKE/0x6300002F
    Received ISAKMP packet: peer = 69.61.228.178
    420    13:12:03.202  12/17/12  Sev=Info/4    IKE/0x63000014
    RECEIVING <<< ISAKMP OAK INFO *(HASH, NOTIFY:DPD_ACK) from 69.61.228.178
    421    13:12:03.202  12/17/12  Sev=Info/5    IKE/0x63000040
    Received DPD ACK from 69.61.228.178, seq# received = 2722476010, seq# expected = 2722476010
    422    13:12:14.185  12/17/12  Sev=Info/4    IKE/0x63000013
    SENDING >>> ISAKMP OAK INFO *(HASH, NOTIFY:DPD_REQUEST) to 69.61.228.178
    423    13:12:14.185  12/17/12  Sev=Info/6    IKE/0x6300003D
    Sending DPD request to 69.61.228.178, our seq# = 2722476011
    424    13:12:14.201  12/17/12  Sev=Info/5    IKE/0x6300002F
    Received ISAKMP packet: peer = 69.61.228.178
    425    13:12:14.201  12/17/12  Sev=Info/4    IKE/0x63000014
    RECEIVING <<< ISAKMP OAK INFO *(HASH, NOTIFY:DPD_ACK) from 69.61.228.178
    426    13:12:14.201  12/17/12  Sev=Info/5    IKE/0x63000040
    Received DPD ACK from 69.61.228.178, seq# received = 2722476011, seq# expected = 2722476011
    427    13:12:24.762  12/17/12  Sev=Info/4    IKE/0x63000013
    SENDING >>> ISAKMP OAK INFO *(HASH, NOTIFY:DPD_REQUEST) to 69.61.228.178
    428    13:12:24.762  12/17/12  Sev=Info/6    IKE/0x6300003D
    Sending DPD request to 69.61.228.178, our seq# = 2722476012
    429    13:12:24.778  12/17/12  Sev=Info/5    IKE/0x6300002F
    Received ISAKMP packet: peer = 69.61.228.178
    430    13:12:24.778  12/17/12  Sev=Info/4    IKE/0x63000014
    RECEIVING <<< ISAKMP OAK INFO *(HASH, NOTIFY:DPD_ACK) from 69.61.228.178
    431    13:12:24.778  12/17/12  Sev=Info/5    IKE/0x63000040
    Received DPD ACK from 69.61.228.178, seq# received = 2722476012, seq# expected = 2722476012
    New running configuration:
    : Saved
    ASA Version 8.4(1)
    hostname NCHCO
    enable password hTjwXz/V8EuTw9p9 encrypted
    passwd hTjwXz/V8EuTw9p9 encrypted
    names
    name 192.168.2.0 NCHCO description City Offices
    name 192.168.2.80 VPN_End
    name 192.168.2.70 VPN_Start
    interface Vlan1
    nameif inside
    security-level 100
    ip address 192.168.2.1 255.255.255.0
    interface Vlan2
    nameif outside
    security-level 0
    ip address 69.61.228.178 255.255.255.248
    interface Ethernet0/0
    switchport access vlan 2
    speed 100
    duplex full
    interface Ethernet0/1
    interface Ethernet0/2
    interface Ethernet0/3
    interface Ethernet0/4
    interface Ethernet0/5
    interface Ethernet0/6
    interface Ethernet0/7
    boot system disk0:/asa841-k8.bin
    ftp mode passive
    object network NCHCO
    subnet 192.168.2.0 255.255.255.0
    object network obj-192.168.1.0
    subnet 192.168.1.0 255.255.255.0
    object network obj-192.168.2.64
    subnet 192.168.2.64 255.255.255.224
    object network obj-0.0.0.0
    subnet 0.0.0.0 255.255.255.0
    object network obj_any
    subnet 0.0.0.0 0.0.0.0
    access-list outside_nat0_outbound extended permit ip object NCHCO 192.168.1.0 255.255.255.0
    access-list outside_nat0_outbound extended permit ip object NCHCO 192.168.2.0 255.255.255.0
    access-list inside_nat0_outbound extended permit ip object NCHCO 192.168.1.0 255.255.255.0
    access-list inside_nat0_outbound extended permit ip any 192.168.2.64 255.255.255.224
    access-list inside_nat0_outbound extended permit ip 0.0.0.0 255.255.255.0 192.168.2.64 255.255.255.224
    access-list outside_1_cryptomap extended permit ip object NCHCO 192.168.1.0 255.255.255.0
    access-list outside_1_cryptomap_1 extended permit ip object NCHCO 192.168.1.0 255.255.255.0
    access-list LAN_Access standard permit 192.168.2.0 255.255.255.0
    access-list LAN_Access standard permit 0.0.0.0 255.255.255.0
    access-list NCHCO_splitTunnelAcl_1 standard permit 192.168.2.0 255.255.255.0
    access-list AnyConnect_Client_Local_Print extended deny ip any any
    access-list AnyConnect_Client_Local_Print extended permit tcp any any eq lpd
    access-list AnyConnect_Client_Local_Print remark IPP: Internet Printing Protocol
    access-list AnyConnect_Client_Local_Print extended permit tcp any any eq 631
    access-list AnyConnect_Client_Local_Print remark Windows' printing port
    access-list AnyConnect_Client_Local_Print extended permit tcp any any eq 9100
    access-list AnyConnect_Client_Local_Print remark mDNS: multicast DNS protocol
    access-list AnyConnect_Client_Local_Print extended permit udp any host 224.0.0.251 eq 5353
    access-list AnyConnect_Client_Local_Print remark LLMNR: Link Local Multicast Name Resolution protocol
    access-list AnyConnect_Client_Local_Print extended permit udp any host 224.0.0.252 eq 5355
    access-list AnyConnect_Client_Local_Print remark TCP/NetBIOS protocol
    access-list AnyConnect_Client_Local_Print extended permit tcp any any eq 137
    access-list AnyConnect_Client_Local_Print extended permit udp any any eq netbios-ns
    pager lines 24
    logging enable
    logging asdm informational
    mtu inside 1500
    mtu outside 1500
    ip local pool VPN_Pool VPN_Start-VPN_End mask 255.255.255.0
    icmp unreachable rate-limit 1 burst-size 1
    asdm image disk0:/asdm-649.bin
    no asdm history enable
    arp timeout 14400
    nat (inside,any) source static NCHCO NCHCO destination static obj-192.168.1.0 obj-192.168.1.0
    nat (inside,any) source static any any destination static obj-192.168.2.64 obj-192.168.2.64
    nat (inside,any) source static obj-0.0.0.0 obj-0.0.0.0 destination static obj-192.168.2.64 obj-192.168.2.64
    object network obj_any
    nat (inside,outside) dynamic interface
    route outside 0.0.0.0 0.0.0.0 69.61.228.177 1
    timeout xlate 3:00:00
    timeout conn 1:00:00 half-closed 0:10:00 udp 0:02:00 icmp 0:00:02
    timeout sunrpc 0:10:00 h323 0:05:00 h225 1:00:00 mgcp 0:05:00 mgcp-pat 0:05:00
    timeout sip 0:30:00 sip_media 0:02:00 sip-invite 0:03:00 sip-disconnect 0:02:00
    timeout sip-provisional-media 0:02:00 uauth 0:05:00 absolute
    timeout tcp-proxy-reassembly 0:01:00
    dynamic-access-policy-record DfltAccessPolicy
    network-acl outside_nat0_outbound
    webvpn
      svc ask enable default svc
    http server enable
    http 192.168.1.0 255.255.255.0 inside
    http 69.61.228.178 255.255.255.255 outside
    http 74.218.158.238 255.255.255.255 outside
    http NCHCO 255.255.255.0 inside
    no snmp-server location
    no snmp-server contact
    snmp-server enable traps snmp authentication linkup linkdown coldstart
    crypto ipsec ikev1 transform-set ESP-AES-128-SHA esp-aes esp-sha-hmac
    crypto ipsec ikev1 transform-set ESP-AES-128-MD5 esp-aes esp-md5-hmac
    crypto ipsec ikev1 transform-set ESP-AES-192-SHA esp-aes-192 esp-sha-hmac
    crypto ipsec ikev1 transform-set ESP-AES-192-MD5 esp-aes-192 esp-md5-hmac
    crypto ipsec ikev1 transform-set ESP-AES-256-SHA esp-aes-256 esp-sha-hmac
    crypto ipsec ikev1 transform-set ESP-AES-256-MD5 esp-aes-256 esp-md5-hmac
    crypto ipsec ikev1 transform-set ESP-3DES-MD5 esp-3des esp-md5-hmac
    crypto ipsec ikev1 transform-set ESP-DES-SHA esp-des esp-sha-hmac
    crypto ipsec ikev1 transform-set ESP-DES-MD5 esp-des esp-md5-hmac
    crypto ipsec ikev1 transform-set l2tp-transform esp-3des esp-sha-hmac
    crypto ipsec ikev1 transform-set l2tp-transform mode transport
    crypto ipsec ikev1 transform-set vpn-transform esp-aes-256 esp-sha-hmac
    crypto ipsec ikev1 transform-set TRANS_ESP_3DES_SHA esp-3des esp-sha-hmac
    crypto ipsec ikev1 transform-set TRANS_ESP_3DES_SHA mode transport
    crypto ipsec ikev1 transform-set TRANS_ESP_3DES_MD5 esp-3des esp-md5-hmac
    crypto ipsec ikev1 transform-set TRANS_ESP_3DES_MD5 mode transport
    crypto ipsec ikev1 transform-set ESP-3DES-SHA esp-3des esp-sha-hmac
    crypto dynamic-map SYSTEM_DEFAULT_CRYPTO_MAP 65535 set pfs group1
    crypto dynamic-map SYSTEM_DEFAULT_CRYPTO_MAP 65535 set ikev1 transform-set ESP-AES-128-SHA ESP-AES-128-MD5 ESP-AES-192-SHA ESP-AES-192-MD5 ESP-AES-256-SHA ESP-AES-256-MD5 ESP-3DES-SHA ESP-3DES-MD5 ESP-DES-SHA ESP-DES-MD5
    crypto dynamic-map dyn-map 10 set pfs group1
    crypto dynamic-map dyn-map 10 set ikev1 transform-set l2tp-transform vpn-transform
    crypto dynamic-map dyn-map 10 set reverse-route
    crypto dynamic-map outside_dyn_map 20 set ikev1 transform-set ESP-3DES-SHA
    crypto map outside_map 1 match address outside_1_cryptomap
    crypto map outside_map 1 set pfs group1
    crypto map outside_map 1 set peer 74.219.208.50
    crypto map outside_map 1 set ikev1 transform-set ESP-3DES-SHA
    crypto map outside_map 20 ipsec-isakmp dynamic outside_dyn_map
    crypto map outside_map 65535 ipsec-isakmp dynamic SYSTEM_DEFAULT_CRYPTO_MAP
    crypto map outside_map interface outside
    crypto map inside_map 65535 ipsec-isakmp dynamic SYSTEM_DEFAULT_CRYPTO_MAP
    crypto map inside_map interface inside
    crypto map vpn-map 1 match address outside_1_cryptomap_1
    crypto map vpn-map 1 set pfs group1
    crypto map vpn-map 1 set peer 74.219.208.50
    crypto map vpn-map 1 set ikev1 transform-set ESP-3DES-SHA
    crypto map vpn-map 10 ipsec-isakmp dynamic dyn-map
    crypto isakmp identity address
    crypto ikev1 enable inside
    crypto ikev1 enable outside
    crypto ikev1 ipsec-over-tcp port 10000
    crypto ikev1 policy 10
    authentication pre-share
    encryption 3des
    hash md5
    group 2
    lifetime 86400
    crypto ikev1 policy 15
    authentication pre-share
    encryption aes-256
    hash sha
    group 2
    lifetime 86400
    crypto ikev1 policy 35
    authentication pre-share
    encryption 3des
    hash sha
    group 2
    lifetime 86400
    client-update enable
    telnet 192.168.1.0 255.255.255.0 inside
    telnet NCHCO 255.255.255.0 inside
    telnet timeout 5
    ssh 192.168.1.0 255.255.255.0 inside
    ssh NCHCO 255.255.255.0 inside
    ssh timeout 5
    console timeout 0
    dhcpd address 192.168.2.150-192.168.2.225 inside
    dhcpd dns 216.68.4.10 216.68.5.10 interface inside
    dhcpd lease 64000 interface inside
    threat-detection basic-threat
    threat-detection statistics access-list
    no threat-detection statistics tcp-intercept
    webvpn
    group-policy DefaultRAGroup internal
    group-policy DefaultRAGroup attributes
    dns-server value 192.168.2.1
    vpn-tunnel-protocol ikev1 l2tp-ipsec
    default-domain value nchco.local
    group-policy DfltGrpPolicy attributes
    dns-server value 192.168.2.1
    vpn-tunnel-protocol ikev1 l2tp-ipsec ssl-client ssl-clientless
    password-storage enable
    ipsec-udp enable
    intercept-dhcp 255.255.255.0 enable
    address-pools value VPN_Pool
    group-policy NCHCO internal
    group-policy NCHCO attributes
    dns-server value 192.168.2.1 8.8.8.8
    vpn-tunnel-protocol ikev1
    split-tunnel-policy tunnelspecified
    split-tunnel-network-list value NCHCO_splitTunnelAcl_1
    default-domain value NCHCO.local
    username admin password LbMiJuAJjDaFb2uw encrypted privilege 15
    username 8njferg password yB1lHEVmHZGj5C2Z encrypted privilege 15
    username NCHvpn99 password dhn.JzttvRmMbHsP encrypted
    tunnel-group DefaultRAGroup general-attributes
    address-pool (inside) VPN_Pool
    address-pool VPN_Pool
    authentication-server-group (inside) LOCAL
    authentication-server-group (outside) LOCAL
    authorization-server-group LOCAL
    authorization-server-group (inside) LOCAL
    authorization-server-group (outside) LOCAL
    default-group-policy DefaultRAGroup
    strip-realm
    strip-group
    tunnel-group DefaultRAGroup ipsec-attributes
    ikev1 pre-shared-key *****
    peer-id-validate nocheck
    tunnel-group DefaultRAGroup ppp-attributes
    no authentication chap
    no authentication ms-chap-v1
    authentication ms-chap-v2
    tunnel-group DefaultWEBVPNGroup ppp-attributes
    authentication pap
    authentication ms-chap-v2
    tunnel-group 74.219.208.50 type ipsec-l2l
    tunnel-group 74.219.208.50 ipsec-attributes
    ikev1 pre-shared-key *****
    tunnel-group NCHCO type remote-access
    tunnel-group NCHCO general-attributes
    address-pool VPN_Pool
    default-group-policy NCHCO
    tunnel-group NCHCO ipsec-attributes
    ikev1 pre-shared-key *****
    class-map inspection_default
    match default-inspection-traffic
    policy-map type inspect dns preset_dns_map
    parameters
      message-length maximum client auto
      message-length maximum 512
    policy-map global_policy
    class inspection_default
      inspect dns preset_dns_map
      inspect ftp
      inspect h323 h225
      inspect h323 ras
      inspect rsh
      inspect rtsp
      inspect esmtp
      inspect sqlnet
      inspect skinny 
      inspect sunrpc
      inspect xdmcp
      inspect sip 
      inspect netbios
      inspect tftp
      inspect ip-options
    service-policy global_policy global
    prompt hostname context
    call-home
    profile CiscoTAC-1
      no active
      destination address http https://tools.cisco.com/its/service/oddce/services/DDCEService
      destination address email [email protected]
      destination transport-method http
      subscribe-to-alert-group diagnostic
      subscribe-to-alert-group environment
      subscribe-to-alert-group inventory periodic monthly
      subscribe-to-alert-group configuration periodic monthly
      subscribe-to-alert-group telemetry periodic daily
    Cryptochecksum:b6ce58676b6aaeba48caacbeefea53a5
    : end
    asdm image disk0:/asdm-649.bin
    asdm location VPN_Start 255.255.255.255 inside
    asdm location VPN_End 255.255.255.255 inside
    no asdm history enable
    I'm at a loss myself as to why this isn't working, and i'm sure that you are running out of solutions yourself. Any other ideas? I really need to get this working.
    Thanks so much!
    Matthew

  • Problem with VPN by ASA 5505 and PIX 501

    Hi
    I have this scenario: Firewall ASA 5505, Firewall Pix 501 (with CatOS 6.3(5) ).
    I have configured this appliance for Easy VPN (server is ASA) and PIX, and remote Access with Cisco client vpn (for internal lan ASA).
    When i configure the ASA i have this problem, when i configure nat for easy vpn.
    This is my nat configuration:
    nat (inside) 0 access-list 100
    nat (inside) 1 192.168.1.0 255.255.255.0
    nat (inside) 1 0.0.0.0 0.0.0.0
    nat (inside) 0 0.0.0.0 0.0.0.0 outside
    when i put this command:
    nat (inside) 0 access-list no-nat
    this command is necessary for configuration of easy vpn, but the previous nat:
    nat (inside) 0 access-list 100
    is replace with the latest command.

    To identify addresses on one interface that are translated to mapped addresses on another interface, use the nat command in global configuration mode. This command configures dynamic NAT or PAT, where an address is translated to one of a pool of mapped addresses. To remove the nat command, use the no form of this command.
    For regular dynamic NAT:
    nat (real_ifc) nat_id real_ip [mask [dns] [outside] [udp udp_max_conns] [norandomseq]]
    no nat (real_ifc) nat_id real_ip [mask [dns] [outside] [udp udp_max_conns] [norandomseq]]
    For policy dynamic NAT and NAT exemption:
    nat (real_ifc) nat_id access-list access_list_name [dns] [outside] [udp udp_max_conns] [norandomseq]
    no nat (real_ifc) nat_id access-list access_list_name [dns] [outside] [udp udp_max_conns] [norandomseq]

  • OWA not accessible after setting up vpn through ASA 5505

    I have a client that is running Win2003 Server R2 with Exchange Server 2003. OWA was setup and clients could connect to their exchange mailbox from the internet with no problems.
    We recently configured vpn on the ASA 5505 and now no-one can connect to OWA since that time.
    Here is what I have for a configuration.
    Any thoughts?

    I thought OWA uses tcp port 80 by default, or 443 if you use https. Ignore the rest if this fact is wrong. Otherwise ...
    In your config there is nothing to allow the www traffic in the access-list outside_access_in
    access-list outside_access_in extended permit tcp any interface outside eq 80
    and no static nat for this
    static (inside,outside) tcp interface 80 192.168.0.254 80 netmask 255.255.255.255
    I think these must have got deleted when you made the other changes ? Hope this helps.

  • Site to Site VPN between ASA 5505 and Cisco 800 router

    Evening all,
    Hoping that someboy can see the error of my ways.  It seems very like the problem that i read here: https://supportforums.cisco.com/thread/2016300
    We have a cisco 800 in a remote site which we wanted to use for a site to site vpn.  Went through the steps on the ASA 5505 and the 800 and have got to the stage were the tunnel is up and connected.  Getting traffic through it is another matter.  Remote network is 172.20.224.0/20 and the server network behind the ASA is 192.168.168.0/24. The tunnel does intiate when you send traffic from 172 ......to 192.......  Both the ASA and 800 report the tunnel is up.  If i look at the stats using ccp on the 800 i can see the encapsulation packets graph shooting up but nothing cominbg back.  I did packet captures on the 5505 and could not see anything coming from the tunnel so i dont belive its making it to the ASA.  Here is the config from the 800:
    Building configuration...
    Current configuration : 6488 bytes
    version 12.4
    no service pad
    service timestamps debug uptime
    service timestamps log uptime
    no service password-encryption
    hostname hhp-sty-backup
    boot-start-marker
    boot-end-marker
    logging message-counter syslog
    logging buffered 4096
    enable secret 5 $1$jI1i$/kZbRk2WHD5h0HtfuQVej1
    aaa new-model
    aaa authentication login default local
    aaa authorization exec default local
    aaa authorization auth-proxy default local
    aaa session-id common
    crypto pki trustpoint TP-self-signed-1347488939
    enrollment selfsigned
    subject-name cn=IOS-Self-Signed-Certificate-1347488939
    revocation-check none
    rsakeypair TP-self-signed-1347488939
    crypto pki certificate chain TP-self-signed-1347488939
    certificate self-signed 02
      30820255 308201BE A0030201 02020102 300D0609 2A864886 F70D0101 04050030
      31312F30 2D060355 04031326 494F532D 53656C66 2D536967 6E65642D 43657274
      69666963 6174652D 31333437 34383839 3339301E 170D3032 30333031 30313336
      33375A17 0D323030 31303130 30303030 305A3031 312F302D 06035504 03132649
      4F532D53 656C662D 5369676E 65642D43 65727469 66696361 74652D31 33343734
      38383933 3930819F 300D0609 2A864886 F70D0101 01050003 818D0030 81890281
      8100E714 7B0ADB41 19F60528 A8A5C43B 5CD2D1CD DCCF2E08 8B38D444 36EAB9B7
      0E93CEF7 660F979E E27915B9 E44812A5 794EA03D BA66752B FD0F7EBF D6342513
      D6410E4E 098CE838 C3BADD0A 5F3505FE 22CA776F 89B19510 F0852225 3600F046
      4D57D2E2 FE4AAD1E 8BE4BF80 7B27369E BFA65160 BC769BC9 00A13741 E336D0EA
      8A810203 010001A3 7D307B30 0F060355 1D130101 FF040530 030101FF 30280603
      551D1104 21301F82 1D686870 2D737479 2D626163 6B75702E 796F7572 646F6D61
      696E2E63 6F6D301F 0603551D 23041830 168014FA 4A8C4DF6 629638DE 87D7B60A
      0F5BB40F EA6AED30 1D060355 1D0E0416 0414FA4A 8C4DF662 9638DE87 D7B60A0F
      5BB40FEA 6AED300D 06092A86 4886F70D 01010405 00038181 00BBE577 6EF63FE7
      789766D5 37841812 298D4885 1CD06D07 4C625369 C3403106 89EE1398 73495432
      66C49CB1 36A5B2F8 D77A8C46 5AFE4112 EA5917D9 81542640 80EF2D36 54A85CC6
      C3FFFFB8 39A648DD 2ABA2B13 4137BE07 760E46C0 74401DA7 482E3FA2 A64B70FF
      447AA1B2 52E37240 29987085 532BBE3B C2E2E54A 54CA1D13 0E
                quit
    dot11 syslog
    ip source-route
    ip dhcp excluded-address 10.10.10.1
    ip dhcp pool inside
    ip dhcp pool lan_network
       network 172.20.224.0 255.255.240.0
       dns-server 8.8.8.8 8.8.4.4
       default-router 172.20.224.1
       lease 7
    ip cef
    no ip domain lookup
    ip domain name yourdomain.com
    password encryption aes
    username pix privilege 15 secret 5 $1$Z.wA$lBmj36AJx/cbK1RjmfGJh1
    username admin privilege 15 password 0 434Zaty
    crypto isakmp policy 1
    encr 3des
    authentication pre-share
    group 2
    crypto isakmp key password address 217.36.32.222
    crypto ipsec transform-set ESP-3DES-SHA esp-3des esp-sha-hmac
    crypto map SDM_CMAP_1 1 ipsec-isakmp
    description Tunnel to217.36.32.222
    set peer 217.36.32.222
    set transform-set ESP-3DES-SHA
    match address 100
    archive
    log config
      hidekeys
    interface ATM0
    no ip address
    no atm ilmi-keepalive
    pvc 0/38
      encapsulation aal5mux ppp dialer
      dialer pool-member 1
    dsl operating-mode auto
    interface FastEthernet0
    interface FastEthernet1
    interface FastEthernet2
    interface FastEthernet3
    interface Vlan1
    description $ETH-SW-LAUNCH$$INTF-INFO-HWIC 4ESW$
    ip address 172.20.224.1 255.255.240.0
    ip nat inside
    ip virtual-reassembly
    ip tcp adjust-mss 1452
    interface Dialer0
    ip address negotiated
    ip nat outside
    ip virtual-reassembly
    encapsulation ppp
    dialer pool 1
    no cdp enable
    ppp authentication chap callin
    ppp chap hostname B6*******.btclick.com
    ppp chap password 0 H*******
    crypto map SDM_CMAP_1
    ip forward-protocol nd
    ip route 0.0.0.0 0.0.0.0 Dialer0
    ip http server
    ip http authentication local
    ip http secure-server
    ip http timeout-policy idle 60 life 86400 requests 10000
    ip nat inside source route-map SDM_RMAP_1 interface Dialer0 overload
    access-list 1 remark CCP_ACL Category=16
    access-list 1 permit 172.4.0.0 0.240.255.255
    access-list 10 permit 195.12.1.35
    access-list 10 permit 172.4.0.0 0.240.255.255
    access-list 100 remark CCP_ACL Category=4
    access-list 100 remark IPSec Rule
    access-list 100 permit ip 172.20.224.0 0.0.15.255 192.168.168.0 0.0.0.255
    access-list 101 remark CCP_ACL Category=2
    access-list 101 remark IPSec Rule
    access-list 101 deny   ip 172.20.224.0 0.0.15.255 192.168.168.0 0.0.0.255
    access-list 101 permit ip 172.4.0.0 0.240.255.255 any
    route-map SDM_RMAP_1 permit 1
    match ip address 101
    control-plane
    banner exec ^C
    % Password expiration warning.
    Cisco Configuration Professional (Cisco CP) is installed on this device
    and it provides the default username "cisco" for  one-time use. If you have
    already used the username "cisco" to login to the router and your IOS image
    supports the "one-time" user option, then this username has already expired.
    You will not be able to login to the router with this username after you exit
    this session.
    It is strongly suggested that you create a new username with a privilege level
    of 15 using the following command.
    username <myuser> privilege 15 secret 0 <mypassword>
    Replace <myuser> and <mypassword> with the username and password you
    want to use.
    ^C
    banner login ^C
    Cisco Configuration Professional (Cisco CP) is installed on this device.
    This feature requires the one-time use of the username "cisco" with the
    password "cisco". These default credentials have a privilege level of 15.
    YOU MUST USE CISCO CP or the CISCO IOS CLI TO CHANGE THESE 
    PUBLICLY-KNOWN CREDENTIALS
    Here are the Cisco IOS commands.
    username <myuser>  privilege 15 secret 0 <mypassword>
    no username cisco
    Replace <myuser> and <mypassword> with the username and password you want
    to use.
    IF YOU DO NOT CHANGE THE PUBLICLY-KNOWN CREDENTIALS, YOU WILL
    NOT BE ABLE TO LOG INTO THE DEVICE AGAIN AFTER YOU HAVE LOGGED OFF.
    For more information about Cisco CP please follow the instructions in the
    QUICK START GUIDE for your router or go to http://www.cisco.com/go/ciscocp
    ^C
    line con 0
    no modem enable
    stopbits 1
    line aux 0
    line vty 0 4
    access-class 10 in
    privilege level 15
    password 434Zaty
    transport input telnet ssh
    scheduler max-task-time 5000
    end
    Any help will be most gratefully recieved.

    Rick,
    Thanks for replying.  Here is the output from the 800 Show Crypto command:
    interface: Dialer0
        Crypto map tag: SDM_CMAP_1, local addr 81.136.160.237
       protected vrf: (none)
       local  ident (addr/mask/prot/port): (172.20.224.0/255.255.240.0/0/0)
       remote ident (addr/mask/prot/port): (192.168.168.0/255.255.255.0/0/0)
       current_peer 217.36.32.222 port 500
         PERMIT, flags={origin_is_acl,}
        #pkts encaps: 10928, #pkts encrypt: 10928, #pkts digest: 10928
        #pkts decaps: 0, #pkts decrypt: 0, #pkts verify: 0
        #pkts compressed: 0, #pkts decompressed: 0
        #pkts not compressed: 0, #pkts compr. failed: 0
        #pkts not decompressed: 0, #pkts decompress failed: 0
        #send errors 2, #recv errors 0
         local crypto endpt.: 81.136.160.237, remote crypto endpt.: 217.36.32.222
         path mtu 1500, ip mtu 1500, ip mtu idb Virtual-Access2
         current outbound spi: 0x0(0)
         inbound esp sas:
         inbound ah sas:
         inbound pcp sas:
         outbound esp sas:
         outbound ah sas:
         outbound pcp sas:
    interface: Virtual-Access2
        Crypto map tag: SDM_CMAP_1, local addr 81.136.160.237
       protected vrf: (none)
       local  ident (addr/mask/prot/port): (172.20.224.0/255.255.240.0/0/0)
       remote ident (addr/mask/prot/port): (192.168.168.0/255.255.255.0/0/0)
       current_peer 217.36.32.222 port 500
         PERMIT, flags={origin_is_acl,}
        #pkts encaps: 10928, #pkts encrypt: 10928, #pkts digest: 10928
        #pkts decaps: 0, #pkts decrypt: 0, #pkts verify: 0
        #pkts compressed: 0, #pkts decompressed: 0
        #pkts not compressed: 0, #pkts compr. failed: 0
        #pkts not decompressed: 0, #pkts decompress failed: 0
        #send errors 2, #recv errors 0
         local crypto endpt.: 81.136.160.237, remote crypto endpt.: 217.36.32.222
         path mtu 1500, ip mtu 1500, ip mtu idb Virtual-Access2
         current outbound spi: 0x0(0)
         inbound esp sas:
         inbound ah sas:
         inbound pcp sas:
         outbound esp sas:
         outbound ah sas:
         outbound pcp sas:
    and this is the running config frm our ASA at HQ:
    Result of the command: "sh run"
    : Saved
    ASA Version 8.2(1)
    hostname secure-access
    domain-name hhp.com
    enable password UWWykvGjAPmxufUo encrypted
    passwd 2KFQnbNIdI.2KYOU encrypted
    names
    interface Vlan1
    nameif inside
    security-level 100
    ip address 192.168.168.1 255.255.255.0
    interface Vlan2
    nameif outside
    security-level 0
    pppoe client vpdn group BT
    ip address 217.36.32.222 255.255.255.255 pppoe
    interface Vlan12
    nameif DMZ
    security-level 50
    ip address 192.168.169.1 255.255.255.0
    interface Vlan22
    nameif Wireless_HHP
    security-level 100
    ip address 172.16.36.1 255.255.254.0
    interface Vlan32
    nameif CNES
    security-level 100
    ip address 187.187.168.1 255.255.0.0
    interface Ethernet0/0
    switchport access vlan 2
    interface Ethernet0/1
    interface Ethernet0/2
    switchport access vlan 12
    interface Ethernet0/3
    switchport access vlan 22
    interface Ethernet0/4
    switchport access vlan 32
    interface Ethernet0/5
    switchport access vlan 12
    interface Ethernet0/6
    switchport access vlan 12
    interface Ethernet0/7
    ftp mode passive
    dns domain-lookup inside
    dns domain-lookup outside
    dns domain-lookup DMZ
    dns domain-lookup Wireless_HHP
    dns domain-lookup CNES
    dns server-group DefaultDNS
    name-server 192.168.168.2
    domain-name hhp.com
    same-security-traffic permit inter-interface
    same-security-traffic permit intra-interface
    object-group network NET-cnes_HHP-Sty
    network-object 172.20.224.0 255.255.240.0
    object-group network NET-cnes_HHP-Balivanich
    network-object 172.20.192.0 255.255.240.0
    object-group network Oak-DC1
    network-object 192.168.168.2 255.255.255.255
    object-group network Maple-DC2
    network-object 192.168.168.3 255.255.255.255
    object-group network HHP_Domain_Controllers
    group-object Oak-DC1
    group-object Maple-DC2
    object-group network PC-Support
    network-object 187.187.60.1 255.255.255.255
    network-object 187.187.60.2 255.255.255.254
    network-object 187.187.60.4 255.255.255.254
    network-object 187.187.60.6 255.255.255.255
    object-group network ELM-ActiveH
    network-object 192.168.168.6 255.255.255.255
    object-group network Pine-GP
    network-object 192.168.168.12 255.255.255.255
    object-group network HHP_Application_Servers
    group-object ELM-ActiveH
    group-object Pine-GP
    object-group network Fern-TS1
    network-object 192.168.168.4 255.255.255.255
    object-group network Fir-TS2
    network-object 192.168.168.5 255.255.255.255
    object-group network HHP_Terminal_Servers
    group-object Fern-TS1
    group-object Fir-TS2
    object-group service Global_Catalog_LDAP
    description (Generated by Cisco SM from Object "Global Catalog LDAP")
    service-object tcp eq 3268
    object-group service Global_Catalog_LDAP_SSL
    description (Generated by Cisco SM from Object "Global Catalog LDAP SSL")
    service-object tcp eq 3269
    object-group service UDP-389
    description UDP port for LDAP
    service-object udp eq 389
    object-group service TCP-88
    description TCP Port 88
    service-object tcp eq 88
    object-group service TCP-445
    description SMB
    service-object tcp eq 445
    object-group network John_-_Laptop
    description John's Laptop
    network-object 187.187.10.65 255.255.255.255
    object-group network Graham_-_PC
    description Graham Morrison's PC
    network-object 187.187.10.90 255.255.255.255
    object-group network john_test
    network-object 187.187.40.7 255.255.255.255
    object-group network Iain_PC
    description Iain Macaulay IT
    network-object 187.187.10.19 255.255.255.255
    object-group network John_-_PC
    description John MacPhail's PC
    network-object 187.187.10.7 255.255.255.255
    object-group network it-alahen-lap
    network-object 187.187.10.230 255.255.255.255
    object-group network Catriona_-_Laptop
    description Catriona's Laptop
    network-object 187.187.10.60 255.255.255.255
    object-group network Graham_-_Laptop
    network-object 187.186.10.120 255.255.255.255
    object-group network it-innive-xp
    description Innes MacIver's PC
    network-object 187.187.10.14 255.255.255.255
    object-group network it-alahen-xp
    description Desktop
    network-object 187.187.10.229 255.255.255.255
    object-group network Cat_-_PC
    description Catriona Macmillan's PC
    network-object 187.187.10.4 255.255.255.255
    object-group network it-davdon-xp
    description Desktop
    network-object 187.187.160.7 255.255.255.255
    object-group network cat-laptop
    description Catriona's Laptop addresses
    network-object 187.187.77.81 255.255.255.255
    network-object 187.187.77.82 255.255.255.255
    object-group network Catriona_old_pc
    network-object 187.187.10.44 255.255.255.255
    object-group network cat-tablet
    description Catriona's Tablet ip address's
    network-object 187.187.77.78 255.255.255.254
    object-group network DSO-SQLServer
    description Task Database Server
    network-object 187.187.1.33 255.255.255.255
    object-group network it-finfernew-xp
    description Findlay Ferguson PC
    network-object 187.187.10.153 255.255.255.255
    object-group network PC_Support
    group-object John_-_Laptop
    group-object Graham_-_PC
    group-object john_test
    group-object Iain_PC
    group-object John_-_PC
    group-object it-alahen-lap
    group-object Catriona_-_Laptop
    group-object Graham_-_Laptop
    group-object it-alahen-xp
    group-object Cat_-_PC
    group-object it-davdon-xp
    group-object cat-laptop
    group-object Catriona_old_pc
    group-object cat-tablet
    group-object it-innive-xp
    network-object 187.187.1.128 255.255.255.255
    network-object 187.187.10.76 255.255.255.255
    group-object DSO-SQLServer
    network-object 187.187.15.234 255.255.255.255
    network-object 187.187.4.60 255.255.255.255
    network-object 187.187.10.134 255.255.255.255
    network-object 172.18.194.22 255.255.255.255
    group-object it-finfernew-xp
    object-group network Entire_CNE
    description Entire CNE range
    network-object 187.0.0.0 255.0.0.0
    object-group network NET-cnes_HHP-Sty-Staff
    network-object 172.20.225.0 255.255.255.0
    object-group network NET-cnes_HHP-Balivanich-staff
    network-object 172.20.193.0 255.255.255.0
    object-group network Alder-Intranet
    network-object 192.168.168.13 255.255.255.255
    object-group network Aspen-ISA
    network-object 192.168.168.10 255.255.255.255
    object-group service tcp-8080
    description TCP Port 8080
    service-object tcp eq 8080
    object-group network Beech-External
    network-object 217.36.32.210 255.255.255.255
    object-group network it-csm
    description cisco security manager
    network-object 187.187.1.72 255.255.255.255
    object-group network Juniper-External
    description Internet Server
    network-object 217.36.32.211 255.255.255.255
    object-group network HHP_Server_Network
    network-object 192.168.168.0 255.255.255.0
    object-group network Messagelabs_Incoming_HHP
    network-object 67.219.240.0 255.255.240.0
    network-object 95.131.104.0 255.255.248.0
    network-object 193.109.254.0 255.255.254.0
    network-object 195.245.230.0 255.255.254.0
    network-object 216.82.240.0 255.255.240.0
    network-object 85.158.136.0 255.255.248.0
    network-object 117.120.16.0 255.255.248.0
    network-object 194.106.220.0 255.255.254.0
    object-group network Angus-Maclean-PC
    network-object 187.187.10.250 255.255.255.255
    object-group service RDP
    service-object tcp eq 3389
    object-group network it-dbserver
    description Database Server (Live)
    network-object 187.187.1.65 255.255.255.255
    object-group network it-sql-test
    description Test SQL / database server
    network-object 187.187.1.81 255.255.255.255
    object-group service DNS-Resolving
    description Domain Name Server
    service-object tcp eq domain
    service-object udp eq domain
    object-group network Beech-Exchange
    network-object 192.168.168.91 255.255.255.255
    object-group network Messagelabs_-_Incoming
    description List of MessageLab addresses that SMTP connections are accepted from
    network-object 212.125.75.0 255.255.255.224
    network-object 216.82.240.0 255.255.240.0
    network-object 195.216.16.211 255.255.255.255
    network-object 194.205.110.128 255.255.255.224
    network-object 194.106.220.0 255.255.254.0
    network-object 193.109.254.0 255.255.254.0
    network-object 62.231.131.0 255.255.255.0
    network-object 62.173.108.208 255.255.255.240
    network-object 62.173.108.16 255.255.255.240
    network-object 212.125.74.44 255.255.255.255
    network-object 195.245.230.0 255.255.254.0
    network-object 85.158.136.0 255.255.248.0
    object-group network MIS_Support
    network-object 192.168.168.250 255.255.255.254
    object-group network it-donadon-xp
    description Donald Macdonald's PC
    network-object 187.187.10.13 255.255.255.255
    object-group network Angela_PC
    network-object 187.187.10.155 255.255.255.255
    object-group network Katie_PC
    network-object 187.187.10.151 255.255.255.255
    object-group network Pauline_PC
    network-object 187.187.10.12 255.255.255.255
    object-group network it-paye-net
    network-object 187.187.1.92 255.255.255.255
    object-group network MessageLabs-Towers
    description Message Labs IP Address ranges
    network-object 216.82.240.0 255.255.240.0
    network-object 67.219.240.0 255.255.240.0
    network-object 85.158.136.0 255.255.248.0
    network-object 95.131.104.0 255.255.248.0
    network-object 117.120.16.0 255.255.248.0
    network-object 193.109.254.0 255.255.254.0
    network-object 194.106.220.0 255.255.254.0
    network-object 195.245.230.0 255.255.254.0
    network-object 62.231.131.0 255.255.255.0
    network-object 212.125.75.16 255.255.255.240
    object-group network NET_cnes-castlebay-staff
    network-object 172.19.17.0 255.255.255.0
    object-group network NET_cnes_tarbert_staff
    description NET_cnes_tarbert_staff
    network-object 172.19.33.0 255.255.255.0
    object-group network Juniper
    network-object 192.168.169.5 255.255.255.255
    object-group network HHP_DMZ_Network
    network-object 192.168.169.0 255.255.255.0
    object-group network Ash
    network-object 192.168.168.100 255.255.255.255
    object-group service UDP-445
    service-object udp eq 445
    object-group service tcp-udp-135-139
    service-object tcp-udp range 135 139
    object-group network HHP-ELM
    description HHP's ELM ActiveH server
    network-object 187.187.1.203 255.255.255.255
    object-group network CNES-Ext-GW
    description CNES External Address
    network-object 194.83.245.242 255.255.255.255
    object-group service IPSEC
    description IPSEC
    service-object 57
    service-object ah
    service-object esp
    service-object udp eq isakmp
    object-group network Alamur-PC
    network-object 187.187.10.15 255.255.255.255
    object-group network Iain-Nicolson-PC
    network-object 187.187.10.159 255.255.255.255
    object-group network HHP_Remote_Access_Pool
    network-object 192.168.168.200 255.255.255.248
    network-object 192.168.168.208 255.255.255.240
    network-object 192.168.168.224 255.255.255.252
    network-object 192.168.168.228 255.255.255.254
    object-group network Holly-AV
    network-object 192.168.168.9 255.255.255.255
    object-group service AVG_Ports
    description For AVG server to HHP PCs
    service-object tcp-udp eq 6150
    service-object tcp-udp eq 6051
    service-object tcp-udp eq 445
    service-object tcp-udp eq 138
    service-object tcp-udp eq 135
    service-object tcp-udp eq 6054
    service-object tcp-udp eq 4158
    service-object tcp-udp eq 139
    service-object tcp-udp eq 137
    object-group network CNES_Access
    network-object 192.168.168.230 255.255.255.254
    network-object 192.168.168.232 255.255.255.248
    network-object 192.168.168.240 255.255.255.248
    network-object 192.168.168.248 255.255.255.254
    object-group network HHP-068
    description BACS PC
    network-object 172.20.225.6 255.255.255.255
    object-group network Banyan
    network-object 192.168.168.105 255.255.255.255
    object-group service TCP81
    description TCP Port 81
    service-object tcp eq 81
    object-group network Gavin_-_new_PC
    network-object 187.187.10.150 255.255.255.255
    object-group network Secudoors
    network-object 172.20.224.4 255.255.255.255
    access-list outside_access_in remark Time sync to external ntp server
    access-list outside_access_in extended permit udp host 192.108.114.23 object-group HHP_Domain_Controllers eq ntp
    access-list outside_access_in extended permit tcp object-group MessageLabs-Towers object-group Beech-External eq smtp
    access-list outside_access_in extended permit ip host 81.136.160.237 object-group HHP_Server_Network
    access-list outside_access_in extended permit ip object-group CNES_Access object-group HHP_Server_Network
    access-list outside_access_in extended permit ip object-group MIS_Support object-group HHP_Server_Network
    access-list outside_access_in extended permit ip object-group HHP_Remote_Access_Pool object-group HHP_Server_Network
    access-list outside_access_in extended permit tcp any object-group Juniper-External eq www
    access-list outside_access_in extended permit tcp any object-group Juniper-External eq https
    access-list outside_access_in extended deny ip any any
    access-list outside_access_in_1 extended permit ip any any
    access-list CSM_FW_ACL_Wireless_HHP extended permit ip object-group NET-cnes_HHP-Balivanich object-group HHP_Server_Network
    access-list CSM_FW_ACL_Wireless_HHP extended permit ip object-group NET-cnes_HHP-Sty object-group HHP_Server_Network
    access-list CSM_FW_ACL_Wireless_HHP extended permit tcp object-group HHP-068 any eq www
    access-list CSM_FW_ACL_Wireless_HHP extended permit tcp object-group HHP-068 any eq domain
    access-list CSM_FW_ACL_Wireless_HHP extended permit udp object-group HHP-068 any eq domain
    access-list CSM_FW_ACL_Wireless_HHP extended permit tcp object-group HHP-068 any eq https
    access-list CSM_FW_ACL_Wireless_HHP extended permit object-group DNS-Resolving object-group HHP-068 any
    access-list CSM_FW_ACL_Wireless_HHP extended permit object-group tcp-8080 object-group HHP-068 any
    access-list CSM_FW_ACL_Wireless_HHP extended permit ip host 172.20.193.53 object-group CNES-Ext-GW
    access-list CSM_FW_ACL_Wireless_HHP extended permit ip object-group Secudoors any
    access-list CSM_FW_ACL_inside extended permit ip object-group HHP_Server_Network object-group NET-cnes_HHP-Balivanich
    access-list CSM_FW_ACL_inside extended permit ip object-group HHP_Server_Network object-group NET-cnes_HHP-Sty
    access-list CSM_FW_ACL_inside extended permit ip object-group HHP_Application_Servers object-group PC_Support
    access-list CSM_FW_ACL_inside extended permit ip object-group HHP_Domain_Controllers object-group PC_Support
    access-list CSM_FW_ACL_inside extended permit ip object-group HHP_Terminal_Servers object-group PC_Support
    access-list CSM_FW_ACL_inside extended permit tcp object-group Oak-DC1 any eq domain
    access-list CSM_FW_ACL_inside extended permit udp object-group Oak-DC1 any eq domain
    access-list CSM_FW_ACL_inside extended permit object-group DNS-Resolving object-group Oak-DC1 any
    access-list CSM_FW_ACL_inside extended permit tcp object-group Maple-DC2 any eq domain
    access-list CSM_FW_ACL_inside extended permit udp object-group Maple-DC2 any eq domain
    access-list CSM_FW_ACL_inside extended permit object-group DNS-Resolving object-group Maple-DC2 any
    access-list CSM_FW_ACL_inside extended permit tcp object-group Aspen-ISA any eq www
    access-list CSM_FW_ACL_inside extended permit tcp object-group Aspen-ISA any eq domain
    access-list CSM_FW_ACL_inside extended permit udp object-group Aspen-ISA any eq domain
    access-list CSM_FW_ACL_inside extended permit tcp object-group Aspen-ISA any eq https
    access-list CSM_FW_ACL_inside extended permit object-group DNS-Resolving object-group Aspen-ISA any
    access-list CSM_FW_ACL_inside extended permit object-group tcp-8080 object-group Aspen-ISA any
    access-list CSM_FW_ACL_inside remark For Symantec Liveupdates
    access-list CSM_FW_ACL_inside extended permit tcp object-group Banyan any eq ftp
    access-list CSM_FW_ACL_inside extended permit tcp object-group Banyan any eq www
    access-list CSM_FW_ACL_inside extended permit tcp object-group Banyan any eq https
    access-list CSM_FW_ACL_inside remark IPSec VPN access from ELm to CNES
    access-list CSM_FW_ACL_inside extended permit object-group IPSEC object-group ELM-ActiveH object-group CNES-Ext-GW
    access-list CSM_FW_ACL_inside extended permit udp object-group ELM-ActiveH object-group CNES-Ext-GW eq 4500
    access-list CSM_FW_ACL_inside extended permit tcp object-group ELM-ActiveH object-group CNES-Ext-GW eq 4500
    access-list CSM_FW_ACL_inside extended permit icmp object-group HHP_Server_Network object-group HHP_DMZ_Network
    access-list CSM_FW_ACL_inside remark Time sync to external ntp server
    access-list CSM_FW_ACL_inside extended permit udp object-group HHP_Domain_Controllers host 192.108.114.23 eq ntp
    access-list CSM_FW_ACL_inside extended permit tcp object-group Beech-Exchange object-group Messagelabs_-_Incoming eq smtp
    access-list CSM_FW_ACL_inside extended permit tcp object-group Aspen-ISA object-group Juniper eq www
    access-list CSM_FW_ACL_inside extended permit tcp object-group Aspen-ISA object-group Juniper eq https
    access-list CSM_FW_ACL_inside extended permit ip object-group Holly-AV object-group Juniper
    access-list CSM_FW_ACL_inside extended deny ip any any
    access-list CSM_FW_ACL_CNES extended permit ip object-group PC_Support object-group HHP_Server_Network
    access-list CSM_FW_ACL_CNES extended permit ip object-group PC_Support object-group HHP_DMZ_Network
    access-list CSM_FW_ACL_CNES extended permit ip object-group PC_Support object-group NET-cnes_HHP-Balivanich
    access-list CSM_FW_ACL_CNES extended permit ip object-group PC_Support object-group NET-cnes_HHP-Sty
    access-list CSM_FW_ACL_CNES extended permit tcp object-group it-csm any eq ssh
    access-list CSM_FW_ACL_CNES extended permit tcp object-group it-csm any eq www
    access-list CSM_FW_ACL_CNES extended permit tcp object-group it-csm any eq https
    access-list CSM_FW_ACL_CNES remark Aim's access to Active H server: DSO SQL
    access-list CSM_FW_ACL_CNES remark server's access (Task)
    access-list CSM_FW_ACL_CNES remark IT Ops - mapped drive for FTP transfer to and from E450/Elm of Entitlement Adjustments
    access-list CSM_FW_ACL_CNES remark and Tenancy Changes
    access-list CSM_FW_ACL_CNES extended permit ip object-group it-sql-test object-group ELM-ActiveH
    access-list CSM_FW_ACL_CNES extended permit ip object-group DSO-SQLServer object-group ELM-ActiveH
    access-list CSM_FW_ACL_CNES extended permit ip object-group it-paye-net object-group ELM-ActiveH
    access-list CSM_FW_ACL_CNES extended permit ip object-group Angela_PC object-group ELM-ActiveH
    access-list CSM_FW_ACL_CNES extended permit ip object-group Katie_PC object-group ELM-ActiveH
    access-list CSM_FW_ACL_CNES extended permit ip object-group Pauline_PC object-group ELM-ActiveH
    access-list CSM_FW_ACL_CNES remark donald and Findlay RDP access to Active H
    access-list CSM_FW_ACL_CNES extended permit object-group RDP object-group it-donadon-xp object-group ELM-ActiveH
    access-list CSM_FW_ACL_CNES extended permit object-group RDP object-group it-donadon-xp object-group HHP_Terminal_Servers
    access-list CSM_FW_ACL_CNES extended permit object-group RDP object-group it-finfernew-xp object-group ELM-ActiveH
    access-list CSM_FW_ACL_CNES extended permit object-group RDP object-group it-finfernew-xp object-group HHP_Terminal_Servers
    access-list CSM_FW_ACL_CNES extended permit ip object-group Angus-Maclean-PC object-group Alder-Intranet
    access-list CSM_FW_ACL_CNES extended permit ip object-group Angus-Maclean-PC host 192.168.168.17
    access-list CSM_FW_ACL_CNES extended permit ip object-group Angus-Maclean-PC object-group Juniper
    access-list CSM_FW_ACL_CNES extended permit ip object-group Iain-Nicolson-PC object-group Alder-Intranet
    access-list CSM_FW_ACL_CNES extended permit ip object-group Iain-Nicolson-PC host 192.168.168.17
    access-list CSM_FW_ACL_CNES extended permit ip object-group Iain-Nicolson-PC object-group Juniper
    access-list CSM_FW_ACL_CNES extended permit ip object-group it-davdon-xp object-group Alder-Intranet
    access-list CSM_FW_ACL_CNES extended permit ip object-group it-davdon-xp host 192.168.168.17
    access-list CSM_FW_ACL_CNES extended permit ip object-group it-davdon-xp object-group Juniper
    access-list CSM_FW_ACL_CNES extended permit ip object-group Alamur-PC object-group Alder-Intranet
    access-list CSM_FW_ACL_CNES extended permit ip object-group Alamur-PC host 192.168.168.17
    access-list CSM_FW_ACL_CNES extended permit ip object-group Alamur-PC object-group Juniper
    access-list CSM_FW_ACL_CNES extended permit ip object-group Gavin_-_new_PC object-group Alder-Intranet
    access-list CSM_FW_ACL_CNES extended permit ip object-group Gavin_-_new_PC host 192.168.168.17
    access-list CSM_FW_ACL_CNES extended permit ip object-group Gavin_-_new_PC object-group Juniper
    access-list CSM_FW_ACL_CNES extended permit object-group RDP object-group NET_cnes-castlebay-staff object-group HHP_Server_Network
    access-list CSM_FW_ACL_CNES extended permit object-group RDP object-group NET_cnes_tarbert_staff object-group HHP_Server_Network
    access-list MIS_splitTunnelAcl standard permit 192.168.168.0 255.255.255.0
    access-list inside_nat0_outbound extended permit ip object-group HHP_Server_Network 192.168.168.250 255.255.255.254
    access-list inside_nat0_outbound extended permit ip object-group HHP_Server_Network 192.168.168.224 255.255.255.224
    access-list CSM_FW_ACL_DMZ extended permit ip object-group HHP_DMZ_Network object-group PC_Support
    access-list CSM_FW_ACL_DMZ extended permit icmp object-group HHP_DMZ_Network object-group HHP_Server_Network
    access-list CSM_FW_ACL_DMZ extended permit ip object-group Juniper object-group Angus-Maclean-PC
    access-list CSM_FW_ACL_DMZ extended permit ip object-group Juniper object-group Holly-AV
    access-list CSM_FW_ACL_DMZ extended permit tcp object-group Juniper object-group Beech-Exchange eq smtp
    access-list CSM_FW_ACL_DMZ extended permit tcp object-group Juniper object-group HHP_Domain_Controllers eq domain
    access-list CSM_FW_ACL_DMZ extended permit udp object-group Juniper object-group HHP_Domain_Controllers eq domain
    access-list CSM_FW_ACL_DMZ remark for backups to USB drive on ASH
    access-list CSM_FW_ACL_DMZ extended permit object-group TCP-445 object-group Juniper object-group Ash
    access-list CSM_FW_ACL_DMZ extended permit object-group UDP-445 object-group Juniper object-group Ash
    access-list CSM_FW_ACL_DMZ extended permit object-group tcp-udp-135-139 object-group Juniper object-group Ash
    access-list CSM_FW_ACL_DMZ extended deny ip any any
    access-list CNES_Support_splitTunnelAcl standard permit 192.168.168.0 255.255.255.0
    access-list RemoteAccess_splitTunnelAcl standard permit 192.168.168.0 255.255.255.0
    access-list outside_cryptomap extended permit ip object-group HHP_Server_Network object-group NET-cnes_HHP-Sty
    pager lines 24
    logging enable
    logging asdm informational
    mtu inside 1500
    mtu outside 1492
    mtu DMZ 1500
    mtu Wireless_HHP 1500
    mtu CNES 1500
    ip local pool CNES_Access 192.168.168.230-192.168.168.249
    ip local pool MIS_Support 192.168.168.250-192.168.168.251
    ip local pool OLM-VPN-Pool 192.168.168.252
    ip local pool HHP_Remote_Access_Pool 192.168.168.200-192.168.168.229
    no failover
    icmp unreachable rate-limit 1 burst-size 1
    no asdm history enable
    arp timeout 14400
    nat-control
    global (outside) 1 interface
    nat (inside) 0 access-list inside_nat0_outbound
    nat (inside) 1 0.0.0.0 0.0.0.0
    nat (Wireless_HHP) 1 172.20.193.53 255.255.255.255
    nat (Wireless_HHP) 1 172.20.225.0 255.255.255.0
    static (inside,CNES) 192.168.168.0 192.168.168.0 netmask 255.255.255.0
    static (CNES,inside) 187.187.0.0 255.255.0.0 netmask 255.255.0.0
    static (Wireless_HHP,inside) 172.20.224.0 172.20.224.0 netmask 255.255.240.0
    static (inside,Wireless_HHP) 192.168.168.0 192.168.168.0 netmask 255.255.255.0
    static (CNES,Wireless_HHP) 187.187.0.0 187.187.0.0 netmask 255.255.0.0
    static (inside,outside) 217.36.32.210 192.168.168.91 netmask 255.255.255.255
    static (DMZ,outside) 217.36.32.211 192.168.169.5 netmask 255.255.255.255
    static (inside,DMZ) 192.168.168.0 192.168.168.0 netmask 255.255.255.0
    static (CNES,DMZ) 187.0.0.0 187.0.0.0 netmask 255.0.0.0
    access-group CSM_FW_ACL_inside in interface inside
    access-group outside_access_in_1 in interface outside control-plane
    access-group outside_access_in in interface outside
    access-group CSM_FW_ACL_DMZ in interface DMZ
    access-group CSM_FW_ACL_Wireless_HHP in interface Wireless_HHP
    access-group CSM_FW_ACL_CNES in interface CNES
    route outside 0.0.0.0 0.0.0.0 81.148.0.157 1
    route Wireless_HHP 172.20.192.0 255.255.240.0 172.16.36.3 1
    route Wireless_HHP 172.20.224.0 255.255.240.0 172.16.36.2 1
    timeout xlate 3:00:00
    timeout conn 1:00:00 half-closed 0:10:00 udp 0:02:00 icmp 0:00:02
    timeout sunrpc 0:10:00 h323 0:05:00 h225 1:00:00 mgcp 0:05:00 mgcp-pat 0:05:00
    timeout sip 0:30:00 sip_media 0:02:00 sip-invite 0:03:00 sip-disconnect 0:02:00
    timeout sip-provisional-media 0:02:00 uauth 0:05:00 absolute
    timeout tcp-proxy-reassembly 0:01:00
    dynamic-access-policy-record DfltAccessPolicy
    aaa-server HHP protocol ldap
    aaa-server HHP (inside) host 192.168.168.2
    timeout 5
    ldap-base-dn dc=hhp,dc=com
    ldap-scope subtree
    ldap-naming-attribute sAMAccountName
    ldap-login-password *
    ldap-login-dn cn=gramor,cn=users,dc=hhp,dc=com
    server-type microsoft
    aaa-server HHP_1 protocol ldap
    aaa-server HHP_1 (inside) host 192.168.168.2
    timeout 5
    ldap-base-dn dc=hhp,dc=com
    ldap-scope subtree
    ldap-naming-attribute sAMAccountName
    ldap-login-password *
    ldap-login-dn cn=administrator,cn=users,dc=hhp,dc=com
    server-type microsoft
    aaa-server HHP_3 protocol ldap
    aaa-server HHP_3 (inside) host 192.168.168.2
    timeout 5
    ldap-base-dn dc=hhp,dc=com
    ldap-scope subtree
    ldap-naming-attribute sAMAccountName
    ldap-login-password *
    ldap-login-dn cn=administrator,cn=users,dc=hhp,dc=com
    server-type microsoft
    aaa authentication ssh console LOCAL
    aaa authentication http console LOCAL
    http server enable
    http 192.168.1.0 255.255.255.0 inside
    http 192.168.168.0 255.255.255.0 inside
    http 0.0.0.0 0.0.0.0 outside
    http 194.83.245.242 255.255.255.255 outside
    http 187.187.1.72 255.255.255.255 CNES
    http 187.187.10.90 255.255.255.255 CNES
    no snmp-server location
    no snmp-server contact
    snmp-server enable traps snmp authentication linkup linkdown coldstart
    crypto ipsec transform-set ESP-AES-256-MD5 esp-aes-256 esp-md5-hmac
    crypto ipsec transform-set ESP-DES-SHA esp-des esp-sha-hmac
    crypto ipsec transform-set ESP-DES-MD5 esp-des esp-md5-hmac
    crypto ipsec transform-set ESP-AES-192-MD5 esp-aes-192 esp-md5-hmac
    crypto ipsec transform-set ESP-3DES-MD5 esp-3des esp-md5-hmac
    crypto ipsec transform-set ESP-AES-256-SHA esp-aes-256 esp-sha-hmac
    crypto ipsec transform-set ESP-AES-128-SHA esp-aes esp-sha-hmac
    crypto ipsec transform-set ESP-AES-192-SHA esp-aes-192 esp-sha-hmac
    crypto ipsec transform-set ESP-AES-128-MD5 esp-aes esp-md5-hmac
    crypto ipsec transform-set ESP-3DES-SHA esp-3des esp-sha-hmac
    crypto ipsec security-association lifetime seconds 28800
    crypto ipsec security-association lifetime kilobytes 4608000
    crypto dynamic-map SYSTEM_DEFAULT_CRYPTO_MAP 65535 set pfs group1
    crypto dynamic-map SYSTEM_DEFAULT_CRYPTO_MAP 65535 set transform-set ESP-AES-128-SHA ESP-AES-128-MD5 ESP-AES-192-SHA ESP-AES-192-MD5 ESP-AES-256-SHA ESP-AES-256-MD5 ESP-3DES-SHA ESP-3DES-MD5 ESP-DES-SHA ESP-DES-MD5
    crypto dynamic-map outside_map_dynamic 2 set transform-set ESP-AES-128-SHA ESP-AES-128-MD5 ESP-AES-192-SHA ESP-AES-192-MD5 ESP-AES-256-SHA ESP-AES-256-MD5 ESP-3DES-SHA ESP-3DES-MD5 ESP-DES-SHA ESP-DES-MD5
    crypto map outside_map 1 match address outside_cryptomap
    crypto map outside_map 1 set peer 81.136.160.237
    crypto map outside_map 1 set transform-set ESP-AES-128-SHA ESP-AES-128-MD5 ESP-AES-192-SHA ESP-AES-192-MD5 ESP-AES-256-SHA ESP-AES-256-MD5 ESP-3DES-SHA ESP-3DES-MD5 ESP-DES-SHA ESP-DES-MD5
    crypto map outside_map 30001 ipsec-isakmp dynamic outside_map_dynamic
    crypto map outside_map 65535 ipsec-isakmp dynamic SYSTEM_DEFAULT_CRYPTO_MAP
    crypto map outside_map interface outside
    crypto ca trustpoint ASDM_TrustPoint0
    enrollment terminal
    fqdn none
    subject-name O=Hebridean Housing Partnership Limited,CN=secure-access.hebrideanhousing.co.uk,L=Isle of Lewis,ST=Scotland,C=GB
    keypair SSL_Certificate
    crl configure
    crypto ca trustpoint ASDM_TrustPoint1
    enrollment terminal
    fqdn none
    crl configure
    crypto ca certificate chain ASDM_TrustPoint0
    certificate 0100000000012790a5c005
        30820530 30820418 a0030201 02020b01 00000000 012790a5 c005300d 06092a86
        4886f70d 01010505 00306a31 23302106 0355040b 131a4f72 67616e69 7a617469
        6f6e2056 616c6964 6174696f 6e204341 31133011 06035504 0a130a47 6c6f6261
        6c536967 6e312e30 2c060355 04031325 476c6f62 616c5369 676e204f 7267616e
        697a6174 696f6e20 56616c69 64617469 6f6e2043 41301e17 0d313030 33323431
        34313835 385a170d 31333033 32343134 31383534 5a308197 310b3009 06035504
        06130247 42311130 0f060355 04081308 53636f74 6c616e64 31163014 06035504
        07130d49 736c6520 6f66204c 65776973 312e302c 06035504 0a132548 65627269
        6465616e 20486f75 73696e67 20506172 746e6572 73686970 204c696d 69746564
        312d302b 06035504 03132473 65637572 652d6163 63657373 2e686562 72696465
        616e686f 7573696e 672e636f 2e756b30 82012230 0d06092a 864886f7 0d010101
        05000382 010f0030 82010a02 82010100 def181d9 c34c58a8 9abcc849 7d8ad0a9
        3c64c77f f3126c81 30911f41 5903a92c 81fb374b 2fe2680e 10b26dce 81ca0c23
        af2c9f9a 52295e8c d2223fa6 7c4c386d 51c6fb16 a47688e6 e47e2410 b0283503
        fd72abd3 e59d3b02 cd47706e babf948c 4e0282a3 5f789ff7 8041b2db ceac64eb
        3e163b38 3a8ecc25 0c4802a8 d17fecd9 f1a36288 29202df4 b20ae891 f95ce055
        6e670559 3d075024 7f3ac7ef 26218154 a7f6a399 34c43c4a 97c2c88c c4588ee4
        77cc2ad8 b1bd868d d55c2b9b 727e9904 66d0fb52 c212abd7 a06f28f1 ad2aa04b
        3d7b3094 c59c00d4 cf51fefb d8bfa101 8ba9c4ba 5cf629ff c50716d3 71019a98
        8fa55b83 6b158b6d 1043f092 646ef07d 02030100 01a38201 a7308201 a3301f06
        03551d23 04183016 80147d6d 2aec66ab a75136ab 0269f170 8fc4590b 9a1f3049
        06082b06 01050507 0101043d 303b3039 06082b06 01050507 3002862d 68747470
        3a2f2f73 65637572 652e676c 6f62616c 7369676e 2e6e6574 2f636163 6572742f
        6f726776 312e6372 74303f06 03551d1f 04383036 3034a032 a030862e 68747470
        3a2f2f63 726c2e67 6c6f6261 6c736967 6e2e6e65 742f4f72 67616e69 7a617469
        6f6e5661 6c312e63 726c301d 0603551d 0e041604 14d398d5 ddf29355 15b04750
        baccc6b3 0f97a6c9 94302f06 03551d11 04283026 82247365 63757265 2d616363
        6573732e 68656272 69646561 6e686f75 73696e67 2e636f2e 756b3009 0603551d
        13040230 00300e06 03551d0f 0101ff04 04030205 a0302906 03551d25 04223020
        06082b06 01050507 03010608 2b060105 05070302 060a2b06 01040182 370a0303
        304b0603 551d2004 44304230 4006092b 06010401 a0320114 30333031 06082b06
        01050507 02011625 68747470 3a2f2f77 77772e67 6c6f6261 6c736967 6e2e6e65
        742f7265 706f7369 746f7279 2f301106 09608648 0186f842 01010404 030206c0
        300d0609 2a864886 f70d0101 05050003 82010100 8af3be01 c4830d83 9b347355
        de7496ef bd76b86c ee92f32f 1157ef11 6ad949b6 611537ad 81f06408 73ec6fe2
        6466675c cf31a80f bead422d ec574f95 55fe0b7a 97e271e7 0220c7b1 53376843
        ff7f7280 f9bfdee6 3584e123 00c37d9f 5004b766 9469ead5 f002744c fd50271c
        6bcdb54c e5db85aa 9760a330 d72464a2 bc8ecdff d80bbc27 7551e97c ee9b7078
        9207f9d6 b969a47a 6df722b6 14ce803d 8d4bb9e9 4695e8e6 d453950e 06506594
        ec7652ea 365cdf94 90e2f7ee 855dadb5 c0459d73 bb6d01a8 3c076718 7f80de40
        c5eb9e0e 17c93087 fd5c5fc1 fd6401fe 7e5038b1 3da1d250 01ccd8be 964d5557
        b320c4c1 0015d1b7 daad7527 930b0c90 7711704f
      quit
    crypto ca certificate chain ASDM_TrustPoint1
    certificate ca 0400000000011e44a5f52a
        30820467 3082034f a0030201 02020b04 00000000 011e44a5 f52a300d 06092a86
        4886f70d 01010505 00305731 0b300906 03550406 13024245 31193017 06035504
        0a131047 6c6f6261 6c536967 6e206e76 2d736131 10300e06 0355040b 1307526f
        6f742043 41311b30 19060355 04031312 476c6f62 616c5369 676e2052 6f6f7420
        4341301e 170d3037 30343131 31323030 30305a17 0d313730 34313131 32303030
        305a306a 31233021 06035504 0b131a4f 7267616e 697a6174 696f6e20 56616c69
        64617469 6f6e2043 41311330 11060355 040a130a 476c6f62 616c5369 676e312e
        302c0603 55040313 25476c6f 62616c53 69676e20 4f726761 6e697a61 74696f6e
        2056616c 69646174 696f6e20 43413082 0122300d 06092a86 4886f70d 01010105
        00038201 0f003082 010a0282 010100a1 2fc4bcce 8703e967 c189c8e5 93fc7db4
        ad9ef663 4e6ae89c 2c7389a2 01f48f21 f8fd259d 58166d86 f6ee4957 757e75ea
        22117e3d fbc74241 dcfcc50c 9155807b eb64331d 9bf9ca38 e9abc625 43512540
        f4e47e18 556aa98f 103a401e d65783ef 7f2f342f 2dd2f653 c2190db7 edc981f5
        462cb423 425e9d13 0375ecea 6afc577c c936973b 98dc1313 ecec41fa 5d34eab9
        93e71016 65cc9c92 fdf5c59d 3e4ab909 fce45f1e 695f4df4 567244b1 1d2303c8
        36f66588 c8bf3916 458e1e26 6c5116c5 2a0038c5 a4136995 7dab013b a8c414b4
        80daac1a 4420d5fe a9067b14 27afe030 21dd90f4 a9d52319 2e1e03e6 c1df9529
        e4c19443 dd3e90aa cb4bc9be 8ad33902 03010001 a382011f 3082011b 300e0603
        551d0f01 01ff0404 03020106 30120603 551d1301 01ff0408 30060101 ff020100
        301d0603 551d0e04 1604147d 6d2aec66 aba75136 ab0269f1 708fc459 0b9a1f30
        4b060355 1d200444 30423040 06092b06 010401a0 32011430 33303106 082b0601
        05050702 01162568 7474703a 2f2f7777 772e676c 6f62616c 7369676e 2e6e6574
        2f726570 6f736974 6f72792f 30330603 551d1f04 2c302a30 28a026a0 24862268
        7474703a 2f2f6372 6c2e676c 6f62616c 7369676e 2e6e6574 2f726f6f 742e6372
        6c301106 09608648 0186f842 01010404 03020204 30200603 551d2504 19301706
        0a2b0601 04018237 0a030306 09608648 0186f842 0401301f 0603551d 23041830
        16801460 7b661a45 0d97ca89 502f7d04 cd34a8ff fcfd4b30 0d06092a 864886f7
        0d010105 05000382 01010079 47fc15d7 4c79df0f 7a9eced4 7c4b63c9 89b57b3f
        9912e89c 8c9a492f e04e954a edc7bcbe f1a2db8e 931dba71 54aa4bd9 89222487
        c504a8ac 8252a052 f8b8e14f a1276663 214a39e7 c7c54e5f b2d61d13 6d30e9ce
        d7a21cbc 290a733c 5b2349fe d6ffcab0 4ff5f267 98c04711 f8b748a6 9009d642
        beeab1b9 5342c39c 20c9fba1 5bb5566d 8781c860 acc4b972 270a8e1e a8b12ecd
        32a27857 b09cf895 bb438e8c 31866e53 0dc61205 ba416ea8 35300918 1d0261ff
        fdee35de 6ac33bd0 4d4b4e50 b256360c 445dda1a 652ae698 56a96333 2e04e7ae
        e8f48eb7 b2da7dc0 c8e2aea6 282fe3c9 73bdfc07 4134b7aa 6eeea7db d1933ced
        90ec3292 88d9c823 6c7421
      quit
    crypto isakmp identity hostname
    crypto isakmp enable outside
    crypto isakmp policy 10
    authentication pre-share
    encryption 3des
    hash sha
    group 2
    lifetime 86400
    crypto isakmp policy 30
    authentication pre-share
    encryption 3des
    hash sha
    group 1
    lifetime 86400
    telnet 0.0.0.0 0.0.0.0 inside
    telnet timeout 5
    ssh 187.187.1.41 255.255.255.255 inside
    ssh 187.187.1.72 255.255.255.255 inside
    ssh 187.187.77.81 255.255.255.255 inside
    ssh 187.187.10.19 255.255.255.255 inside
    ssh 187.187.10.229 255.255.255.255 inside
    ssh 187.187.160.7 255.255.255.255 inside
    ssh 187.187.1.41 255.255.255.255 outside
    ssh 187.187.1.72 255.255.255.255 outside
    ssh 187.187.77.81 255.255.255.255 outside
    ssh 187.187.10.19 255.255.255.255 outside
    ssh 187.187.10.229 255.255.255.255 outside
    ssh 187.187.160.7 255.255.255.255 outside
    ssh timeout 15
    console timeout 0
    vpdn group BT request dialout pppoe
    vpdn group BT localname B*******.btclick.com
    vpdn group BT ppp authentication chap
    vpdn username B*******@hg39.btclick.com password *********
    dhcpd auto_config outside
    threat-detection basic-threat
    threat-detection statistics access-list
    no threat-detection statistics tcp-intercept
    ssl trust-point ASDM_TrustPoint0 outside
    ssl trust-point ASDM_TrustPoint0 outside vpnlb-ip
    webvpn
    enable inside
    enable outside
    group-policy HHP_Remote_Access_1 internal
    group-policy HHP_Remote_Access_1 attributes
    wins-server value 192.168.168.2 192.168.168.2
    dns-server value 192.168.168.2 192.168.168.3
    vpn-tunnel-protocol IPSec
    split-tunnel-policy tunnelspecified
    split-tunnel-network-list value CNES_Support_splitTunnelAcl
    group-policy HHP_Remote_Access internal
    group-policy HHP_Remote_Access attributes
    wins-server value 192.168.168.2 192.168.168.2
    dns-server value 192.168.168.2 192.168.168.3
    vpn-tunnel-protocol IPSec
    split-tunnel-policy tunnelspecified
    split-tunnel-network-list value CNES_Support_splitTunnelAcl
    group-policy Omfax internal
    group-policy Omfax attributes
    wins-server value 192.168.168.2 192.168.168.3
    dns-server value 192.168.168.2 192.168.168.3
    vpn-tunnel-protocol IPSec webvpn
    webvpn
      svc ask none default webvpn
    group-policy MIS_1 internal
    group-policy MIS_1 attributes
    wins-server value 192.168.168.2 192.168.168.3
    dns-server value 192.168.168.2 192.168.168.3
    vpn-tunnel-protocol IPSec
    split-tunnel-policy tunnelspecified
    split-tunnel-network-list value MIS_splitTunnelAcl
    default-domain value hhp.com
    group-policy RemoteAccess internal
    group-policy RemoteAccess attributes
    wins-server value 192.168.168.2 192.168.168.3
    dns-server value 192.168.168.2 192.168.168.3
    vpn-tunnel-protocol IPSec
    split-tunnel-policy tunnelspecified
    split-tunnel-network-list value RemoteAccess_splitTunnelAcl
    group-policy CNES_Access internal
    group-policy CNES_Access attributes
    wins-server value 192.168.168.2 192.168.168.3
    dns-server value 192.168.168.2 192.168.168.3
    vpn-tunnel-protocol IPSec
    split-tunnel-policy tunnelspecified
    split-tunnel-network-list value CNES_Support_splitTunnelAcl
    group-policy HHP internal
    group-policy HHP attributes
    dhcp-network-scope none
    vpn-access-hours none
    vpn-idle-timeout none
    vpn-session-timeout none
    vpn-filter none
    vpn-tunnel-protocol IPSec webvpn
    password-storage disable
    ip-comp disable
    re-xauth disable
    group-lock none
    pfs disable
    ipsec-udp disable
    split-tunnel-policy tunnelall
    split-tunnel-network-list none
    split-dns none
    secure-unit-authentication disable
    user-authentication disable
    user-authentication-idle-timeout none
    ip-phone-bypass disable
    leap-bypass disable
    nem disable
    backup-servers keep-client-config
    client-firewall none
    webvpn
      url-list value Severs
      filter none
      homepage none
      port-forward disable
      http-proxy disable
      sso-server none
      svc dtls none
      svc keep-installer none
      svc rekey time none
      svc rekey method none
      svc dpd-interval client none
      svc dpd-interval gateway none
      svc compression none
      svc modules none
      svc profiles none
      svc ask none default webvpn
      customization none
      http-comp none
      user-storage none
      storage-key none
      hidden-shares none
      smart-tunnel disable
      activex-relay disable
      file-entry disable
      file-browsing disable
      url-entry disable
      deny-message none
    group-policy MIS internal
    group-policy MIS attributes
    wins-server value 192.168.168.2 192.168.168.3
    dns-server value 192.168.168.2 192.168.168.3
    vpn-tunnel-protocol IPSec
    split-tunnel-policy tunnelspecified
    split-tunnel-network-list value MIS_splitTunnelAcl
    username test password Kg/Rgy23do7gPGTv encrypted privilege 0
    username test attributes
    vpn-group-policy HHP_Remote_Access
    username catneil password yOgiHCGobUNIkjcN encrypted privilege 0
    username omfax password pvUaCLwilGmQVifd encrypted privilege 0
    username backup password IHQbl.JAoESlM9Jv encrypted privilege 0
    username misadmin password 8IZXmHa67HIJYHK1 encrypted
    username misadmin attributes
    service-type remote-access
    username gramor password ne829U0rGFVEedhY encrypted privilege 15
    username gramor attributes
    vpn-group-policy HHP_Remote_Access
    webvpn
      url-list value Severs
    username aim_user password 5OQaWCdB18qiHlOn encrypted privilege 0
    username aim_user attributes
    vpn-group-policy CNES_Support
    username katask password 2WsX.HoqKXuiqkDk encrypted privilege 0
    username katask attributes
    vpn-group-policy CNES_Support
    username janboyd password ZEUyykwzME6hII2i encrypted privilege 0
    username marmor password C5n48AiRLXwxAeBQ encrypted privilege 0
    username marste password amwTL584WdiT87Tb encrypted privilege 0
    username helmah password RvU8c.3w0H3/MJz4 encrypted privilege 0
    username anglea password wGlUJDBrmJI.uz./ encrypted privilege 0
    username anglea attributes
    vpn-group-policy CNES_Support
    username fiobuc password 5Uispw90wqvDYerQ encrypted privilege 0
    tunnel-group DefaultRAGroup general-attributes
    authentication-server-group HHP_1
    tunnel-group DefaultWEBVPNGroup general-attributes
    authentication-server-group HHP_1
    default-group-policy HHP
    tunnel-group DefaultWEBVPNGroup webvpn-attributes
    nbns-server 192.168.168.2 timeout 2 retry 2
    nbns-server 192.168.168.3 timeout 2 retry 2
    tunnel-group WebVPN type remote-access
    tunnel-group WebVPN general-attributes
    authentication-server-group HHP_3
    default-group-policy HHP
    username-from-certificate UID
    tunnel-group CNES_Access

  • Smart Card Problem with AnyConnect over RDP

    Hello,
    For ASA 5545, v-8.6(1)2 and AnyConnect v-3.1.0165, I'm trying to start an AnyConnect client tunnel on a remote RDP (both ends Windows 7) machine and am having problems. The RDP is configured to proxy smart card devices which generally works fine. I'm using current SafeNet eToken with current client software. When I start AnyConnect from client machines (no RDP), the tunnel opens with no problem using the smart card. When I try to start the tunnel on the remote machine via RDP, I'm prompted for cert selection and smart card PIN, but get a popup from AnyConnect: 'VPN connection terminated, smart card removed from reader'. When I try to start the tunnel via RDP but use the ASA web server to start, the tunnel starts up fine with the smart card.
    For the problem condition, the Windows event log on the remote RDP machine shows 3 entires (see below) wrt acvpnagent show smart card removal errors but the USB device is always inserted. Also, in investigating, I changed the client profile 'server list' config to SSL instead of IPSec. Same failure but the popup does not show.
    VPN connection terminated, Smartcard removed from reader.
    Description: VPNMGR_ERROR_SMARTCARD_REMOVED:A smartcard required for the connection has been removed
    Thanks in advance for any assistance.
    Mike

    This can be resolved by going to services and disabling Smart card

  • Routing issue with 2 VPN on ASA

    Hello,
    I am trying to setup a VPN between 3 sites :
    site2 and site3 needs to communicate with site1(ASA) :
         site1(ASA)
         |               |
         |               |
    site2          site3
             Peer
    On site2 / site3 if have multiple peers that want to communicate to site 1 and that can arrive indifferently on site2 or site3 firewall.
    All VPNs are UP but there is a routing problem is located on the ASA. Indeed, site2 to site1 communication is ok in both directions. The problem comes from site3.
    On site3, incoming packets reach the target on site1 through the VPN, but the answer is sent back through site1/site2 VPN.
    Is there a simple way to force the trafic to use the same VPN for responding data ?
    Here is a sample of the configuration on the ASA (subnet on site2 and site3 must be left on 'any') :
    access-list outside_cryptomap_1 extended permit ip 10.0.0.0 255.255.0.0 any
    access-list outside_cryptomap_2 extended permit ip 10.0.0.0 255.255.0.0 any
    crypto map my-crypto-map 1 match address outside_cryptomap_1
    crypto map my-crypto-map 1 set pfs
    crypto map my-crypto-map 1 set peer 90.X.Y.Z
    crypto map my-crypto-map 1 set transform-set ESP-AES-256-SHA
    crypto map my-crypto-map 1 set security-association lifetime kilobytes 51200
    crypto map my-crypto-map 2 match address outside_cryptomap_2
    crypto map my-crypto-map 2 set pfs
    crypto map my-crypto-map 2 set peer 190.X.Y.Z
    crypto map my-crypto-map 2 set transform-set ESP-AES-256-SHA

    No, this is not possible, you cannot have overlapping crypto ACLs.
    One possible solution might be to apply NAT to the traffic before it enters the tunnel on site3.
    But so this requires changing from "any" to one or more specific networks.
    hth
    Herbert

  • Setup vpn in ASA 5505

    We have a hosted server with a new provider and we also opted for a firewall which is a ASA 5505. It turns out that they do not provide assistance with the firewall, so I have come here!
    The server hosts multiple customer websites, along with both MySQL and SQL Server databases.
    I would like to provide customers with the ability to connect to their databases remotely, for which I assume we need to create a VPN to allow them access through the firewall. Also, I would want them to be able to use the standard Windows connection tool(s).
    Unfortunately, I know very little about this so I am looking for idiot-proof help ;-)
    First of all, is this a safe thing to do - can I control what they can access on the server end?
    What type of VPN do I need? I looked at one of the wizards and stopped at the point when it started talking about IP addresses ... what range should I / can I use? How does that relate to our server setup, or doesn't it?
    I don't know what else to ask ... hopefully you can guide me in the right direction!

    Can you add this:
    tunnel-group 64.53.58.229 type ipsec-l2l
    tunnel-group 64.53.58.229 ipsec-attributes
    pre-shared-key
    Another thing i have noticed is the disconnect between the WAN IP (VLAN 2) and default gateway next hop IP address.
    nterface Vlan2
    nameif inside
    security-level 100
    ip address 204.116.85.166 255.255.255.0
    route outside 0.0.0.0 0.0.0.0 207.144.35.1 1
    Please rate replies and mark question as "answered" if applicable.

  • Unable to establish site to site vpn between asa 5505 an 5510

    Hi ALL expert
    We are now plan to form a site to site IPSec VPN tunnel between ASA 5505 (ASA Version 8.4) and ASA 5510 (ASA Version 8.0) but failure, would you please teach me how to establish it? Any reference guide?
    Hugo

    Here are the links to the cisco config-guides:
    http://www.cisco.com/en/US/docs/security/asa/asa80/configuration/guide/site2sit.html
    http://www.cisco.com/en/US/docs/security/asa/asa84/configuration/guide/vpn_site2site.html
    In addition to VPN you need to look into NAT exemption:
    http://www.cisco.com/en/US/docs/security/asa/asa80/configuration/guide/cfgnat.html#wp1043541
    http://www.cisco.com/en/US/docs/security/asa/asa84/configuration/guide/nat_overview.html#wpxref25608
    http://www.cisco.com/en/US/docs/security/asa/asa84/configuration/guide/nat_rules.html#wp1232160
    And lots of examples:
    http://www.cisco.com/en/US/products/ps6120/prod_configuration_examples_list.html
    Don't stop after you've improved your network! Improve the world by lending money to the working poor:
    http://www.kiva.org/invitedby/karsteni

  • Slow connection with Anyconnect VPN

    Hi,
    I have ASA 5510 and FW.8.4. I created Anyconnect VPN but connection is very slow even though internet speed is 100Mb up and down. Users cannot run applications since they time out. Please see the result of "sh vpn-session svc".
    Than you for your help in advance.
    sh vpn-session svc
    Session Type: AnyConnect
    Username     : Test                   Index        : 13
    Assigned IP  : 192.168.145.1          Public IP    : 89.29.1xx.xxx
    Protocol     : AnyConnect-Parent SSL-Tunnel DTLS-Tunnel
    License      : AnyConnect Premium
    Encryption   : RC4 RC4 AES128         Hashing      : SHA1 SHA1 SHA1
    Bytes Tx     : 29066337               Bytes Rx     : 982348
    Group Policy : GroupPolicy_MY-REAder  Tunnel Group : MF-Ridder
    Login Time   : 14:00:54 CEST Thu Dec 18 2014
    Duration     : 0h:08m:09s
    Inactivity   : 0h:00m:00s
    NAC Result   : Unknown
    VLAN Mapping : N/A                    VLAN         : none

    Check this link to upgade or reflash the router's firmware.

  • Problem with Cisco VPN since upgrade to Snow Leopard

    Since upgrading to Snow Leopard a couple of hours ago I can no longer open up my CiscoVPN application (version 4.9.01) which I use to connect to my network at work. When I try now I just get an "Error 51: message"
    *Error 51: Unable to communicate with the VPN subsystem.*
    Please make sure that you have at least one network interface that is currently active and has an IP address and start this application again.
    I can still connect to the network if I use Parallels and use the Windows VPN client - but that's not really the point! I've tried the popular suggestion for Error 51 errors in the past (typing "sudo SystemStarter restart CiscoVPN" via Terminal) which doesn't work.
    Any ideas or help appreciated.
    Acculess

    I was able to get it working without re-downloading the client (4.9.0.1.0080 was already installed). I noticed that System/Library/StartupItems/CiscoVPN had been removed during the upgrade to snow leopard, so I pulled it back off of a Time Machine backup. Restoring that directory and doing a 'sudo /System/Library/StartupItems/CiscoVPN/CiscoVPN restart' seemed to fix things up.

Maybe you are looking for

  • Error in Form 16 standard service.

    Hi All, When i try to access the form 16 standard service i am facing an exception called Error when calling method INIT of class CL_HRXSS_HR_IN_F16 . Please do the needful. We do not have NWDI setup in our organisation. I sthere any other way or pat

  • FCP 5.1.4 on G5 does not start

    HI All, Yesterday I have been working on a project in FCP and this morning FCP does not start. After selection it shows me the little box (the startup box) but after 6 seconds it stops. I have checked the console and see this NOTE: ignoring this comp

  • Security Exception from Java stand-alone program on Linux

    Hi All, I am getting the following exception while running a Java stand-alone program on Linux. The stand-alone program internally calls the JCE (Java Cryptography Extension) library for Encryption of data. Does anybody have the solution for this err

  • Themes in the workbook

    Hi Gurus,        I want to set the themes in my workbook.        Pls give me some steps to do it. With Regards, Yokesh.

  • How to NOT log certain cronjobs

    I have a job that runs once a minute, and it logs the event to /var/log/crond.  This makes for huge and boring log files.  Is there a way to turn off logging for just this job?  Is there an option I can set in my crontab?